SlideShare a Scribd company logo
1 of 182
Download to read offline
GMAT Critical Reasoning - Everything you need to know
This slideshow features screenshots from GMAT Prep Now’s
entire Critical Reasoning module (consisting of 38 videos).
It covers everything you need to know, and it includes 24
practice questions.
www.GMATPrepNow.com
www.GMATPrepNow.com
Note: since these slides are just snippets of a full-length video
course, there may be times when you’re unable to glean all the
relevant information from a particular screenshot.
If, at any time, you’d like to watch the entire video on a certain
topic, just click on the link at the top of that page, and you’ll be
taken that that particular video.
GMAT Critical Reasoning - Everything you need to know
If you enjoy this learning format, let us
know, and we’ll add similar resources to
our SlideShare page
GMAT Critical Reasoning - Everything you need to know
Introduction to Critical Reasoning (watch the entire video here)
This lesson features GMAT questions from the GMAT® mini-test.
GMAT® questions are the property of the Graduate Management
Admission Council® and are used here with their permission.
Introduction to Critical Reasoning (watch the entire video here)
• About 1/3 of Verbal questions (13 to 15 questions)
• Batches of 2 or 3 questions
• Approximately 2 minutes per question
• Test your ability to reason effectively
Introduction to Critical Reasoning (watch the entire video here)
Introduction to Critical Reasoning
The average normal infant born in the United
States weighs between twelve and fourteen
pounds at the age of three months. Therefore,
if a three-month-old child weighs only ten
pounds, its weight gain has been below the
United States average.
Which of the following indicates a flaw in the
reasoning above?
A) Weight is only one measure of normal
infant development.
B) Some three-month-old children weigh as
much as seventeen pounds.
C) It is possible for a normal child to weigh
ten pounds at birth.
D) The phrase "below average" does not
necessarily mean insufficient.
E) Average weight gain is not the same as
average weight.
Passage
(watch the entire video here)
Introduction to Critical Reasoning
The average normal infant born in the United
States weighs between twelve and fourteen
pounds at the age of three months. Therefore,
if a three-month-old child weighs only ten
pounds, its weight gain has been below the
United States average.
Which of the following indicates a flaw in the
reasoning above?
A) Weight is only one measure of normal
infant development.
B) Some three-month-old children weigh as
much as seventeen pounds.
C) It is possible for a normal child to weigh
ten pounds at birth.
D) The phrase "below average" does not
necessarily mean insufficient.
E) Average weight gain is not the same as
average weight.
Passage
Question stem
(watch the entire video here)
Introduction to Critical Reasoning
The average normal infant born in the United
States weighs between twelve and fourteen
pounds at the age of three months. Therefore,
if a three-month-old child weighs only ten
pounds, its weight gain has been below the
United States average.
Which of the following indicates a flaw in the
reasoning above?
A) Weight is only one measure of normal
infant development.
B) Some three-month-old children weigh as
much as seventeen pounds.
C) It is possible for a normal child to weigh
ten pounds at birth.
D) The phrase "below average" does not
necessarily mean insufficient.
E) Average weight gain is not the same as
average weight.
Passage
Question stem
Answer choices
(watch the entire video here)
Introduction to Critical Reasoning
• About 1/3 of Verbal questions (13 to 15 questions)
• Batches of 2 or 3 questions
• Approximately 2 minutes per question
• Test your ability to reason effectively
• Arguments consist of premises and a conclusion
• Conclusion: what the author is trying to convince you of
• Premises: the evidence used to support the conclusion
(watch the entire video here)
Introduction to Critical Reasoning
Premise: All men are mortal
Premise: Socrates is a man
All men are mortal. Socrates is a man.
Therefore, Socrates is mortal.
Conclusion: Socrates is mortal
+
Premise: Gary late for last 3 days
Premise: Flat tire this morning
Conclusion: Gary will be late today
+
Strength of an Argument: How well the conclusion follows
from the premises
Inductive: the conclusion
is not guaranteed
Deductive: the conclusion
is guaranteed
For the past 3 days, Gary has arrived late
for work. This morning, while Gary was
driving to work, his car got a flat tire.
Therefore, Gary will be late for work today.
(watch the entire video here)
Introduction to Critical Reasoning
Strength of an Argument: How well the conclusion follows
from the premises
Strengthen
Strengthen
For the past 3 days, Gary has arrived late
for work. This morning, while Gary was
driving to work, his car got a flat tire.
Therefore, Gary will be late for work today.
For the past 53 days, Gary has arrived
late for work. This morning, while Gary
was driving to work, his car got a flat tire.
Therefore, Gary will be late for work today.
For the past 53 days, Gary has arrived
late for work. This morning, while Gary
was driving to work, his car got 4 flat
tires, and his engine exploded.
Therefore, Gary will be late for work today.
(watch the entire video here)
Introduction to Critical Reasoning
Strength of an Argument: How well the conclusion follows
from the premises
Weaken
For the past 3 days, Gary has arrived late
for work. This morning, while Gary was
driving to work, his car got a flat tire.
Therefore, Gary will be late for work today.
For the past 3 days, Gary has arrived late
for work. This morning, while Gary was
driving to work, his car got a flat tire.
Gary’s house is 1 block from work.
Therefore, Gary will be late for work today.
(watch the entire video here)
Introduction to Critical Reasoning
Strength of an Argument: How well the conclusion follows
from the premises
• The conclusions in most arguments are not guaranteed
• Most arguments can be strengthened or weakened by
adding/altering premises
For the past 3 days, Gary has arrived late
for work. This morning, while Gary was
driving to work, his car got a flat tire.
Therefore, Gary will be late for work today.
(watch the entire video here)
Introduction to Critical Reasoning
Strength of an Argument
How well the conclusion follows
from the premises
Accept all premises as true!
For the past 3 days, Gary has arrived late
for work. This morning, while Gary was
driving to work, his car got a flat tire.
Therefore, Gary will be late for work today.
(watch the entire video here)
Introduction to Critical Reasoning
All pigs can fly. Mount Everest is a pig.
Therefore, Mount Everest can fly.
Accept all premises as true!

(watch the entire video here)
Introduction to Critical Reasoning
All pigs can fly. Mount Everest is a pig.
Therefore, Mount Everest cannot fly.
Accept all premises as true!

(watch the entire video here)
Introduction to Critical Reasoning
• About 1/3 of Verbal questions (13 to 15 questions)
• Batches of 2 or 3 questions
• Approximately 2 minutes per question
• Test your ability to reason effectively
• Arguments consist of premises and a conclusion
• Conclusion: what the author is trying to convince you of
• Premises: the evidence used to support the conclusion
• Assumption(s): unstated premise(s) necessary to reach conclusion
(watch the entire video here)
Introduction to Critical Reasoning
Assumptions: • Gary’s history of lateness will be
repeated today
• The time needed to change tire will
exceed the time before work starts
• Gary has no other means by which to
arrive at work on time
For the past 3 days, Gary has arrived late
for work. This morning, while Gary was
driving to work, his car got a flat tire.
Therefore, Gary will be late for work today.
(watch the entire video here)
Introduction to Critical Reasoning
Premise(s)
Conclusion
+Assumption(s)
(watch the entire video here)
Introduction to Critical Reasoning
• About 1/3 of Verbal questions (13 to 15 questions)
• Batches of 2 or 3 questions
• Approximately 2 minutes per question
• Test your ability to reason effectively
• Arguments consist of premises and a conclusion
• Conclusion: what the author is trying to convince you of
• Premises: the evidence used to support the conclusion
• Assumption(s): unstated premise(s) necessary to reach conclusion
Strength of an Argument: How well the conclusion follows
from the premises
Accept all premises as true!
(watch the entire video here)
Dissecting an Argument (watch the entire video here)
Dissecting an Argument
Every hockey fan I know is nice. I do not
know Judy, but since she is wearing a
hockey jersey, she must be nice.
Summarize:
- the conclusion
- the premises
- any assumptions
(watch the entire video here)
Dissecting an Argument
“Is this what the
author is trying to
convince me of?”
Every hockey fan I know is nice. I do not
know Judy, but since she is wearing a
hockey jersey, she must be nice.
Conclusion: J is nice
(watch the entire video here)
Dissecting an Argument
Every hockey fan I know is nice. I do not
know Judy, but since she is wearing a
hockey jersey, she must be nice.
Goal: make your summaries as brief as
possible (while maintaining coherence)
Conclusion: J is nice
(watch the entire video here)
Dissecting an Argument
Conclusion: J is nice
Every hockey fan I know is nice. I do not
know Judy, but since she is wearing a
hockey jersey, she must be nice.
Premise: All H-fans I know are nice
Premise: I don’t know J
Premise: J wearing jersey
+
Assumption: The H-fans I know are representative
Assumption: Wearing jersey makes one a fan
(watch the entire video here)
Dissecting an Argument
Researcher: Two years ago, a wolf pack was
relocated to Bilford Island. Although the local
rabbit population has decreased drastically
since the relocation, the wolves are not to
blame for this decrease. Our study shows that
the unprecedented number of recent rabbit
deaths is due to the myxoma virus.
Summarize:
- the conclusion
- the premises
- any assumptions
(watch the entire video here)
Dissecting an Argument
Premise-Therefore-Conclusion test
Researcher: Two years ago, a wolf pack was
relocated to Bilford Island. Although the local
rabbit population has decreased drastically
since the relocation, the wolves are not to
blame for this decrease. Our study shows that
the unprecedented number of recent rabbit
deaths is due to the myxoma virus.
Option A: wolves not to blame therefore virus killed rabbits
Option B: virus killed rabbits therefore wolves not to blame 

(watch the entire video here)
Dissecting an Argument
Conclusion:  rabbit pop. not wolves’ fault
+
Researcher: Two years ago, a wolf pack was
relocated to Bilford Island. Although the local
rabbit population has decreased drastically
since the relocation, the wolves are not to
blame for this decrease. Our study shows that
the unprecedented number of recent rabbit
deaths is due to the myxoma virus.
Premise: wolves arrived 2 yrs ago
Premise: rabbit pop.  since
Premise: virus caused deaths
Assumption: wolves didn’t contribute to virus
(watch the entire video here)
Dissecting an Argument
Tips for Identifying Conclusions and Premises
• Watch for trigger words that indicate a conclusion
• Watch for trigger words that indicate a premise
therefore, thus, hence, so, implies, indicates, consequently,
as a result, clearly, accordingly, infer, conclude
since, because, for, due to, evidence, on the basis of,
given that
• Beware of common argument structures
1. Premise, premise, . . . , conclusion
2. Conclusion, premise, premise . . .
3. Conclusion in the question stem
(watch the entire video here)
Dissecting an Argument
Summarize:
- the conclusion
- the premises
- any assumptions
Until recently, the only fish species living in Chilliwack Lake
was the Gigafish. Last month, however, several Sovkafish were
spotted in the lake. Unlike Gigafish, Sovkafish do not eat
insects; instead, they survive by eating other fish. In other
lakes where Sovkafish exist, their populations are limited by
Dragonfish, which like to feed on Sovkafish.
Which of the following, if true, most effectively challenges the
conclusion that releasing 100 Dragonfish into Chilliwack Lake
will allow the Gigafish in Chilliwack Lake to survive?
(watch the entire video here)
Dissecting an Argument
Until recently, the only fish species living in Chilliwack Lake
was the Gigafish. Last month, however, several Sovkafish were
spotted in the lake. Unlike Gigafish, Sovkafish do not eat
insects; instead, they survive by eating other fish. In other
lakes where Sovkafish exist, their populations are limited by
Dragonfish, which like to feed on Sovkafish.
Which of the following, if true, most effectively challenges the
conclusion that releasing 100 Dragonfish into Chilliwack Lake
will allow the Gigafish in Chilliwack Lake to survive?
Conclusion: Releasing Dragons will let Gigas live
+
Premise: Giga were only fish in lake
Premise: Sovkas now in lake
Premise: Sovkas eat other fish
Premise: Dragons eat Sovkas
Assumption: Dragons won't eat the Gigas
Assumption: Dragons won't somehow jeopardize Gigas
Assumption: 100 Dragons is sufficient
(watch the entire video here)
Dissecting an Argument
Premise-Therefore-Conclusion test
Tips for Identifying Conclusions and Premises
• Watch for trigger words that indicate a conclusion
• Watch for trigger words that indicate a premise
therefore, thus, hence, so, implies, indicates, consequently,
as a result, clearly, accordingly, infer, conclude
since, because, for, due to, evidence, on the basis of,
given that
• Beware of common argument structures
1. Premise, premise, . . . , conclusion
2. Conclusion, premise, premise . . .
3. Conclusion in the question stem
(watch the entire video here)
Common Argument Types (watch the entire video here)
Common Argument Types
3 Most Common Argument Types
• Cause and Effect
• Statistical
• Analogy
(watch the entire video here)
Common Argument Types
Cause and Effect
Conclusion: X causes Y
+
Premise: Event X occurs
Premise: Event Y occurs
Assumption: X is the only possible cause of Y
A recent study reveals that the rate of obesity is
higher among senior citizens who watch more than 8
hours of television per day than among senior citizens
who watch fewer than 8 hours of television per day.
Therefore, obesity among senior citizens is caused by
watching more than 8 hours of television per day.
correlation ≠ causation
Weaken
• Something else causes Y
• Y causes X
• X and Y are coincidental
Strengthen
• More information
• Eliminate other causes of Y
(watch the entire video here)
Common Argument Types
Statistical
Conclusion: Something about entire population
+
Premise: Information from sample
Assumption: Sample represents entire population
In a recent survey, participants at a Republicans-
only dance competition were given a questionnaire.
Most of the respondents indicated that they enjoyed
singing. Therefore, it can be concluded that most
Republicans are outgoing people.
Weaken
• Sample not representative
• Conclusion doesn’t match stats
• Flaw in calculations
Strengthen
• Sample is representative
(watch the entire video here)
Common Argument Types
Analogy
Conclusion: Some other similarity exists
+
Premise: Similarity between X and Y
Premise: Similarity between X and Y
Premise: Similarity between X and Y
Assumption: Sharing some means sharing all
Country X is a democratic, tropical country with a population
of 5 million, and Country Y is a democratic, tropical country
with a population of 5 million. Since Country X is
experiencing widespread crop failures, Country Y must be
experiencing widespread crop failures as well.
Weaken
• Entities less similar
Strengthen
• Entities even more similar
(watch the entire video here)
Common Argument Types
Cause and Effect
Conclusion: X causes Y
+
Premise: Event X occurs
Premise: Event Y occurs
Assumption: X is the only possible cause of Y
Weaken
• Something else causes Y
• Y causes X
• X and Y are coincidental
Strengthen
• More information
• Eliminate other causes of Y
Statistical
Conclusion: Something about entire population
+
Premise: Information from sample
Assumption: Sample represents entire population
Weaken
• Sample not representative
• Conclusion doesn’t match stats
• Flaw in calculations
Strengthen
• Sample is representative
Analogy
Conclusion: Some other similarity exists
+
Premise: Similarity between X and Y
Premise: Similarity between X and Y
Premise: Similarity between X and Y
Assumption: Sharing some means sharing all
Weaken
• Entities less similar
Strengthen
• Entities even more similar
(watch the entire video here)
General Critical Reasoning Strategy (watch the entire video here)
Question Types
General Critical Reasoning Strategy
1. Weaken the Argument
2. Strengthen the Argument
3. Assumption
4. Conclusion/Inference
5. Method of Reasoning
6. Flawed Argument
7. Paradox
8. Evaluation
(watch the entire video here)
General Strategy
General Critical Reasoning Strategy
1. Read the question stem to determine the question type
2. Read the passage (argument) and focus on the required
information for that question type
3. Check all answer choices
(watch the entire video here)
Weaken the Argument Questions (watch the entire video here)
Weaken the Argument Questions
Conclusion
Premise
Premise
Premise
Assumption
Assumption+
(A) New Premise
(B) New Premise
(C) New Premise
(D) New Premise
(E) New Premise
Goal: Find the answer choice that, when added to
the argument, undermines the conclusion the most.
(watch the entire video here)
Weaken the Argument Questions
General Strategy
1. Read the question stem to determine the question type
2. Read the passage (argument) and focus on the required
information for that question type
3. Check all answer choices
(watch the entire video here)
Weaken the Argument Questions
Question stem examples
Which of the new premises hurts the argument the most?
• Which of the following, if true, most seriously weakens the
argument above?
• Which of the following, if true, would cast the most serious
doubt on the researcher’s conclusion?
• Which of the following, if true, most effectively challenges
the conclusion that releasing 100 Dragonfish into Chilliwack
Lake will allow the Gigafish in Chilliwack Lake to survive?
• Which of the following, if established, most seriously calls into
question the argument above?
• Which of the following, if true, most seriously undermines the
spokesperson’s position?
(watch the entire video here)
Weaken the Argument Questions
General Strategy
1. Read the question stem to determine the question type
2. Read the passage (argument) and focus on the required
information for that question type
3. Check all answer choices
(watch the entire video here)
Weaken the Argument Questions
Weaken the Argument Strategy
1. Identify and summarize the conclusion
2. Identify and summarize the premises
3. Identify any assumptions
4. Check each answer choice while
repeating conclusion
5. Check all answer choices
Does this weaken
the conclusion
that...?
(watch the entire video here)
Weaken the Argument Questions
Tips
- Undermining an unstated assumption
- Adding a new premise that hurts the conclusion
1. Look for common argument types (cause and effect,
statistical, analogy)
2. Common ways to weaken an argument
3. “Weaken” does not necessarily mean “destroy”
4. Beware of answer choices that strengthen the argument
5. Do not try to disprove a premise
6. Goal: Weaken the extent to which the conclusion follows
from the premises
(watch the entire video here)
Weaken the Argument Questions
Weaken the Argument Strategy
1. Identify and summarize the conclusion
2. Identify and summarize the premises
3. Identify any assumptions
4. Check each answer choice while
repeating conclusion
5. Check all answer choices
Example conclusion: Eating carrots improves one’s eyesight
Does this weaken
the conclusion
that...?
(watch the entire video here)
Practice Question
A program instituted in a particular state allows parents to prepay their
children's future college tuition at current rates. The program then pays
the tuition annually for the child at any of the state's public colleges in
which the child enrolls. Parents should participate in the program as a
means of decreasing the cost of their children's college education.
Which of the following, if true, is the most appropriate reason for
parents not to participate in the program?
A. The parents are unsure about which public college in
the state the child will attend.
B. The amount of money accumulated by putting the
prepayment funds in an interest-bearing account
today will be greater than the total cost of tuition for
any of the public colleges when the child enrolls.
C. The annual cost of tuition at the state's public
colleges is expected to increase at a faster rate than
the annual increase in the cost of living.
D. Some of the state's public colleges are contemplating
large increases in tuition next year.
E. The prepayment plan would not cover the cost of
room and board at any of the state's public colleges.
This is a question from the
GMAT® mini-test. GMAT®
questions are the property of the
Graduate Management Admission
Council® and are used here with
their permission.
Practice Question
A program instituted in a particular state allows parents to prepay their
children's future college tuition at current rates. The program then pays
the tuition annually for the child at any of the state's public colleges in
which the child enrolls. Parents should participate in the program as a
means of decreasing the cost of their children's college education.
Which of the following, if true, is the most appropriate reason for
parents not to participate in the program?
P: Program: prepay
tuition @
current rates
P: Program pays
tuition later
C: Program is
cheaper
A: Today’s cost less
than future cost
A. The parents are unsure about which public college in
the state the child will attend.
B. The amount of money accumulated by putting the
prepayment funds in an interest-bearing account
today will be greater than the total cost of tuition for
any of the public colleges when the child enrolls.
C. The annual cost of tuition at the state's public
colleges is expected to increase at a faster rate than
the annual increase in the cost of living.
D. Some of the state's public colleges are contemplating
large increases in tuition next year.
E. The prepayment plan would not cover the cost of
room and board at any of the state's public colleges.




(watch the entire video here)
Practice Question
To attract members away from other fitness clubs in the city, FitnessTown
recently launched a new marketing campaign in which each new member
receives a free locker rental, free fitness consultations, and unlimited use
of the tanning beds. FitnessTown’s marketing team claims that the
campaign has been a huge success, since its membership has already
increased 20 percent in the first two weeks of the campaign.
Which of the following, if true, would best help to refute the claim of
FitnessTown’s marketing team?
A. Almost all of FitnessTown’s new members had never
visited a fitness club before they were enticed by
FitnessTown’s new campaign.
B. Most of FitnessTown’s new members do not intend to
use the free services offered in the campaign.
C. FitnessTown’s investors hope to increase membership
by 50 percent within the first month of the campaign.
D. FitnessTown’s new marketing campaign is identical
to its previous marketing campaigns.
E. Studies show that people typically cancel their fitness
club memberships within the first year.
Practice Question
To attract members away from other fitness clubs in the city, FitnessTown
recently launched a new marketing campaign in which each new member
receives a free locker rental, free fitness consultations, and unlimited use
of the tanning beds. FitnessTown’s marketing team claims that the
campaign has been a huge success, since its membership has already
increased 20 percent in the first two weeks of the campaign.
Which of the following, if true, would best help to refute the claim of
FitnessTown’s marketing team?
A. Almost all of FitnessTown’s new members had never
visited a fitness club before they were enticed by
FitnessTown’s new campaign.
B. Most of FitnessTown’s new members do not intend to
use the free services offered in the campaign.
C. FitnessTown’s investors hope to increase membership
by 50 percent within the first month of the campaign.
D. FitnessTown’s new marketing campaign is identical
to its previous marketing campaigns.
E. Studies show that people typically cancel their fitness
club memberships within the first year.
P: Campaign =
locker, consult.
& tanning
P: Membership up
20% in 2 wks
C: FitnessTown
stole others’
members
A: membership
increase means
stolen
customers.




(watch the entire video here)
Strengthen the Argument Questions (watch the entire video here)
Conclusion
Premise
Premise
Premise
Assumption
Assumption+
(A) New Premise
(B) New Premise
(C) New Premise
(D) New Premise
(E) New Premise
Goal: Find the answer choice that, when added to
the argument, strengthens the conclusion the most.
Strengthen the Argument Questions (watch the entire video here)
Question stem examples
Which of the new premises improves the
argument the most?
Strengthen the Argument Questions
• Which of the following, if true, would provide the most
support for the conclusion of the argument above?
• Which of the following statements, if true, most strengthens
the author’s argument?
• Which of the following, if true, provides the best indication
that Suki’s decision was logically sound?
• Which of the following, if true, most strongly supports the
recommendation made by the argument?
(watch the entire video here)
Does this strengthen
the conclusion
that...?
Example conclusion: Antonio makes the world’s best spaghetti
Strengthen the Argument Strategy
Strengthen the Argument Questions
1. Identify and summarize the conclusion
2. Identify and summarize the premises
3. Identify any assumptions
4. Check each answer choice while
repeating conclusion
5. Check all answer choices
(watch the entire video here)
Tips
• Look for common argument types (cause and effect,
statistical, analogy)
• Common ways to strengthen an argument
• The goal is not to create a perfect argument
• Beware of answer choices that weaken the argument
• Watch out for answer choices that support a premise but
not the conclusion
Strengthen the Argument Questions
- Stating a previously-unstated assumption
- Supporting or elaborating on an existing premise
- Adding a new supporting premise
Bonnie must be great chess player
Avram is world-class sprinter
Avram is great chess player
Bonnie is world-class sprinter+
(A) Bonnie holds 100m world record
(B) .
(C) .
(D) .
(E) .

(watch the entire video here)
Strengthen the Argument Questions
Does this strengthen
the conclusion
that...?
Strengthen the Argument Strategy
1. Identify and summarize the conclusion
2. Identify and summarize the premises
3. Identify any assumptions
4. Check each answer choice while
repeating conclusion
5. Check all answer choices
(watch the entire video here)
Practice Question
In the 12 years since the pretzel desalinization plant was built on the
Polahoochi River, the salinity of the river has increased from 180 µS/cm to 911
µS/cm in the 10-mile stretch of river downstream from the plant. During the
same 12-year period, researchers have observed a sharp decline in the
average number of eggs laid by individual polafish, a species native to the
Polahoochi River. Given this information, Dr. Kim hypothesizes that high
salinity levels adversely affect the reproductive organs of the polafish.
Which of the following, if true, would most strengthen Dr. Kim’s hypothesis?
A. In the same state, many populations of polafish that
are not downstream of pretzel desalinization plants
have experienced a reduction in egg production.
B. Prior to the construction of the pretzel desalinization
plant, the salinity of the 10-mile stretch of river
downstream never exceeded 180 µS/cm.
C. Other species of fish in the same 10-mile stretch have
experienced a sharp reduction in egg production.
D. In the past 12 years, the salinity of the tributaries
flowing into the 10-mile stretch of river downstream
from the plant has remained below 180 µS/cm, and
the polafish living in these tributaries have not
experienced any decline in egg production.
E. In other states, fish downstream from pretzel desalinization
plants have experienced declines in egg production.
Practice Question
In the 12 years since the pretzel desalinization plant was built on the
Polahoochi River, the salinity of the river has increased from 180 µS/cm to 911
µS/cm in the 10-mile stretch of river downstream from the plant. During the
same 12-year period, researchers have observed a sharp decline in the
average number of eggs laid by individual polafish, a species native to the
Polahoochi River. Given this information, Dr. Kim hypothesizes that high
salinity levels adversely affect the reproductive organs of the polafish.
Which of the following, if true, would most strengthen Dr. Kim’s hypothesis?
P: plant blt 12 yrs ago
P: downstream: salt 
P: downstream:
polafish eggs 
C:  salinity lowers
polafish egg #s
A:  salinity is only
possible cause of
egg 
A. In the same state, many populations of polafish that
are not downstream of pretzel desalinization plants
have experienced a reduction in egg production.
B. Prior to the construction of the pretzel desalinization
plant, the salinity of the 10-mile stretch of river
downstream never exceeded 180 µS/cm.
C. Other species of fish in the same 10-mile stretch have
experienced a sharp reduction in egg production.
D. In the past 12 years, the salinity of the tributaries
flowing into the 10-mile stretch of river downstream
from the plant has remained below 180 µS/cm, and
the polafish living in these tributaries have not
experienced any decline in egg production.
E. In other states, fish downstream from pretzel desalinization
plants have experienced declines in egg production.




(watch the entire video here)
Assumption Questions (watch the entire video here)
Conclusion
Premise
Premise
Premise
Assumption
Assumption+
(A) Assumption
(B) Assumption
(C) Assumption
(D) Assumption
(E) Assumption
Goal: Find a necessary assumption
Assumption Questions (watch the entire video here)
Question stem examples
Assumption Questions
• Which of the following is an assumption on which the
argument depends?
• The scientist’s argument depends on the assumption that
• The conclusion above follows logically if which of the
following is assumed?
(watch the entire video here)
Is this assumption
necessary to draw the
conclusion that...?
Assumption Question Strategy
1. Identify and summarize the conclusion
2. Identify and summarize the premises
3. Identify any assumptions
4. Look for one of your assumptions among
the answer choices
5. Check each answer choice against the
conclusion
Assumption Questions
Example conclusion: Hotdogs are bad for one’s health
(watch the entire video here)
Strengthen the Argument Questions
Assumption Questions
Assumption Questions
Want information that helps the conclusion
Want information that is necessary for the conclusion
(watch the entire video here)
Negation Technique
Assumption Questions
Basis: 1) An assumption is absolutely necessary for
a conclusion to follow from the premises
2) Negating a necessary assumption will destroy
the argument
Juan has been practicing tennis 3 hours each day for the past 2
years. Therefore, Juan will win the city championship next month.
C: J will win championship
P: J practicing 3hr/day for 2 yrs
+
A: Nothing stops championship
A: J is eligible to play
A: It is not the case that J lives until championship
A: J lives until championship
P: J dies before championship
P: J dies before championship


(watch the entire video here)
Is this assumption
necessary to draw the
conclusion that...?
Assumption Question Strategy
1. Identify and summarize the conclusion
2. Identify and summarize the premises
3. Identify any assumptions
4. Look for one of your assumptions among
the answer choices
5. Check each answer choice against the
conclusion
6. Apply Negation Technique:
7. Check all answer choices
Assumption Questions
Negate each answer choice and insert it into the argument.
The negated answer choice that destroys the argument is
the correct answer
(watch the entire video here)
Tips
• Look for common argument types (cause and effect,
statistical, analogy)
• Look for shifts in language between premises and
conclusion
• Remember that arguments can have any number of
assumptions
Assumption Questions
Juan has been practicing tennis 3 hours each day for the past 2
years. Therefore, Juan will win the city championship next month.
C: J will win championship
P: J practicing 3hr/day for 2 mths
+
A: J lives until championship
A: Nothing stops championship
A: J is eligible to play
(watch the entire video here)
Practice Question
A. Investment firms should not restrict the
ways in which portfolio managers
manage their clients’ funds.
B. Portfolio managers can provide above-
average returns for their corporate
clients only by short selling stocks.
C. Short selling is a technique used
primarily for corporate clients.
D. Portfolio managers often used short
selling techniques to provide above-
average returns for corporate clients.
E. Before the investment firm issued the
new rules, portfolio managers were not
permitted to short sell stocks.
A leading investment firm has issued new rules that prevent its portfolio
managers from short selling stocks for their corporate clients. One
portfolio manager has concluded that this restriction prevents portfolio
managers from providing above-average investment returns for their
corporate clients.
Which of the following is an assumption that would allow the portfolio
manager’s conclusion to be properly drawn?
Practice Question
A. Investment firms should not restrict the
ways in which portfolio managers
manage their clients’ funds.
B. Portfolio managers can provide above-
average returns for their corporate
clients only by short selling stocks.
C. Short selling is a technique used
primarily for corporate clients.
D. Portfolio managers often used short
selling techniques to provide above-
average returns for corporate clients.
E. Before the investment firm issued the
new rules, portfolio managers were not
permitted to short sell stocks.
A leading investment firm has issued new rules that prevent its portfolio
managers from short selling stocks for their corporate clients. One
portfolio manager has concluded that this restriction prevents portfolio
managers from providing above-average investment returns for their
corporate clients.
Which of the following is an assumption that would allow the portfolio
manager’s conclusion to be properly drawn?
A: SS necessary to make big $




P: New rules prevent SS
C: Rules stop big $ for CC
(watch the entire video here)
Conclusion/Inference Questions (watch the entire video here)
?
Premise
Premise
Premise+
(A) Conclusion
(B) Conclusion
(C) Conclusion
(D) Conclusion
(E) Conclusion
Goal: Find conclusion that logically follows
Conclusion/Inference Questions (watch the entire video here)
Question stem examples
Conclusion/Inference Questions
• The statements above, if true, most strongly support which of
the following conclusions?
• If the statements above are true, which of the following must
also be true on the basis of them?
• Which of the following hypotheses receives the strongest
support from the given information?
• Which of the following can be logically inferred based on the
statements above?
Identify something that must follow from the premises
Inference question = Conclusion question
(watch the entire video here)
Conclusion/Inference Questions
For the past 3 days, all of Florida’s orange farms have
experienced freezing temperatures. Therefore, the number
of oranges harvested this year will be less than expected.
Typical Conclusion (in most GMAT questions)
Conclusion in a Conclusion question
• Conclusion is partially supported
For the past 3 days, the temperature at every Florida orange
farm has not exceeded -5 degrees Celsius.
The statement above, if true, most strongly supports which of
the following conclusions?
• Conclusion is guaranteed
(A) The number of oranges harvested this year will be less
than expected.

(watch the entire video here)
Conclusion/Inference Questions
For the past 3 days, all of Florida’s orange farms have
experienced freezing temperatures. Therefore, the number
of oranges harvested this year will be less than expected.
Typical Conclusion (in most GMAT questions)
Conclusion in a Conclusion question
• Conclusion is partially supported
For the past 3 days, the temperature at every Florida orange
farm has not exceeded -5 degrees Celsius.
The statement above, if true, most strongly supports which of
the following conclusions?
• Conclusion is guaranteed
(A) The orange harvest at some Florida farms will be less
than expected.

(watch the entire video here)
Conclusion/Inference Questions
For the past 3 days, all of Florida’s orange farms have
experienced freezing temperatures. Therefore, the number
of oranges harvested this year will be less than expected.
Typical Conclusion (in most GMAT questions)
Conclusion in a Conclusion question
• Conclusion is partially supported
For the past 3 days, the temperature at every Florida orange
farm has not exceeded -5 degrees Celsius.
The statement above, if true, most strongly supports which of
the following conclusions?
• Conclusion is guaranteed
(A) At least one orange will be damaged from the freezing
temperatures.

(watch the entire video here)
Conclusion/Inference Questions
For the past 3 days, all of Florida’s orange farms have
experienced freezing temperatures. Therefore, the number
of oranges harvested this year will be less than expected.
Typical Conclusion (in most GMAT questions)
Conclusion in a Conclusion question
• Conclusion is partially supported
For the past 3 days, the temperature at every Florida orange
farm has not exceeded -5 degrees Celsius.
The statement above, if true, most strongly supports which of
the following conclusions?
• Conclusion is guaranteed
(A) For the past 3 days, not one Florida orange farm has
experienced temperatures above -5 degrees Celsius.

(watch the entire video here)
Conclusion/Inference Questions
For the past 3 days, all of Florida’s orange farms have
experienced freezing temperatures. Therefore, the number
of oranges harvested this year will be less than expected.
Typical Conclusion (in most GMAT questions)
Conclusion in a Conclusion question
• Conclusion is partially supported
For the past 3 days, the temperature at every Florida orange
farm has not exceeded -5 degrees Celsius.
The statement above, if true, most strongly supports which of
the following conclusions?
• Conclusion is guaranteed
(A) During the past 3 days, at least one Florida orange farm has
experienced temperatures lower than 40 degrees Celsius.

(watch the entire video here)
Must it be true
that...?
Conclusion Question Strategy
1. Identify and summarize the premises
2. Draw a conclusion that must follow
3. Look for your conclusion among the
answer choices
4. Aggressively eliminate incorrect answers
5. Apply a version of the Negation Technique:
6. Check all answer choices
Conclusion/Inference Questions
The negated conclusion that contradicts the premises the
most is probably the correct answer.
(watch the entire video here)
Tips
1. Do not stray too far from the premises
2. Look for a rewording of a premise
P: Kyle enjoys chocolate
P: Rome is the capital of Italy
Conclusion/Inference Questions
Example
C: ?
What conclusion can be drawn
using every premise?

(watch the entire video here)
Tips
1. Do not stray too far from the premises
2. Look for a rewording of a premise
P: Kyle enjoys chocolate
P: Rome is the capital of Italy
Conclusion/Inference Questions
Example
C: The capital of Italy is Rome 
(watch the entire video here)
Tips
1. Do not stray too far from the premises
2. Look for a rewording of a premise
3. Conclusions need not involve every premise
4. Do not inject assumptions into the argument
5. Beware of answer choices that introduce new ideas/words
6. Beware of answer choices where the strength of the language
does not match the strength of the language in the premises
Conclusion/Inference Questions (watch the entire video here)
Practice Question
While studying Emperor Penguins in Antarctica during the coldest 3 months
of the year, researchers observed that several of the penguins died. The
researchers also discovered that the death rate among the larger penguins
was greater than the death rate among the smaller penguins. However, the
researchers’ conclusion that size was a determinant in these deaths is probably
mistaken, since smaller penguins are typically younger than larger ones.
If the above statements are true, which of the following can be inferred?
A. Among Emperor Penguins of the same age,
the smaller penguins have a greater survival
rate over a 3-month period than the larger
penguins do.
B. Among the Emperor Penguins that survived
the 3-month period, there is probably no
relationship between age and size.
C. Among Emperor Penguins of the same age, a
difference in size may not indicate a difference
in chances of survival over a 3-month period.
D. Temperature does not play a role in the death
rate among Emperor Penguins over a 3-month
period.
E. Among Emperor Penguins of the same size,
age is a determinant in the survival rate over
a 3-month period.
Practice Question
While studying Emperor Penguins in Antarctica during the coldest 3 months
of the year, researchers observed that several of the penguins died. The
researchers also discovered that the death rate among the larger penguins
was greater than the death rate among the smaller penguins. However, the
researchers’ conclusion that size was a determinant in these deaths is probably
mistaken, since smaller penguins are typically younger than larger ones.
If the above statements are true, which of the following can be inferred?
P: P’s died during 3 coldest mnths
P: Larger P’s more likely to die
P: Resrchrs conclude size a factor
P: Size probably not a factor
P: Smaller P’s typically younger
A. Among Emperor Penguins of the same age,
the smaller penguins have a greater survival
rate over a 3-month period than the larger
penguins do.
B. Among the Emperor Penguins that survived
the 3-month period, there is probably no
relationship between age and size.
C. Among Emperor Penguins of the same age, a
difference in size may not indicate a difference
in chances of survival over a 3-month period.
D. Temperature does not play a role in the death
rate among Emperor Penguins over a 3-month
period.
E. Among Emperor Penguins of the same size,
age is a determinant in the survival rate over
a 3-month period.




(watch the entire video here)
Structure Questions (watch the entire video here)
Structure Questions
• Test your understanding of the argumentative strategies
employed in an argument
• 3 types of Structure Questions:
- Method of Reasoning
- Boldface
- Parallel Argument
(watch the entire video here)
(A) Description of argument
(B) Description of argument
(C) Description of argument
(D) Description of argument
(E) Description of argument
Goal: Find the best description of the author’s
argumentative strategy.
Method of Reasoning questions
Conclusion
Premise
Premise
Premise
Assumption
Assumption+
Structure Questions (watch the entire video here)
Question stem examples for Method of Reasoning questions
Explain how the author presents his/her argument
Structure Questions
• The author’s point is made by which method of reasoning?
• Which of the following strategies does Dr. Kwan use to
defend his position?
• In the passage, the author develops the argument by ___
• The reporter challenges the spokesperson’s position by
doing which of the following?
(watch the entire video here)
Examples of answer choices for Method of Reasoning questions
Structure Questions
• The argument arrives at its conclusion by demonstrating
the inherent problems with alternative conclusions.
• The author offers a new definition of a term that is central
to an opposing argument.
• The argument employs circular reasoning by assuming
that which it is trying to prove.
The answer choices are typically generic
(watch the entire video here)
Strategy for tackling Method of Reasoning questions
1. Read the passage
2. For each sentence, ask, “What role does this play in
the argument?”
3. Identify and summarize the conclusion and premises
4. Use generic language to describe the method of
reasoning to yourself
5. Look for your description among the answer choices
6. Check all answer choices
Structure Questions (watch the entire video here)
(A) Role played by boldfaced portion(s)
(B) Role played by boldfaced portion(s)
(C) Role played by boldfaced portion(s)
(D) Role played by boldfaced portion(s)
(E) Role played by boldfaced portion(s)
Goal: Find the best description of the role(s) played
Boldface questions
Conclusion
Premise
Premise
Premise
Assumption
Assumption+
Structure Questions (watch the entire video here)
Question stem examples for Boldface questions
The passage contains bolded text
Structure Questions
• In the above argument, the portion in boldface plays
which of the following roles?
• In the researcher’s argument, the two portions in
boldface play which of the following roles?
Researcher: Two years ago, a wolf pack was
relocated to Bilford Island. Although the local
rabbit population has decreased drastically
since the relocation, the wolves are not to
blame for this decrease. Our study shows
that the unprecedented number of recent
rabbit deaths is due to the myxoma virus.
In the above argument, the portion in boldface
plays which of the following roles?
(watch the entire video here)
Strategy for tackling Boldface questions
1. Read the passage
2. For each boldfaced portion, ask, “What role does this
play in the argument?”
3. Identify and summarize the conclusion and premises
4. Use generic language to describe the roles played by the
boldfaced portion(s)
5. Find the answer choice that most closely matches yours
6. Check all answer choices
Structure Questions (watch the entire video here)
Tips for Boldface questions
1. Look for common roles:
2. Consider how the second bolded part is related to
first bolded part
3. Beware of answer choices that are half right and half
wrong
Structure Questions
- Concluding
- Summarizing
- Contradicting
- Providing supporting evidence
- Providing an example
- Providing a counterexample
- Generalizing
(watch the entire video here)
Structure Questions
(A) Complete argument
(B) Complete argument
(C) Complete argument
(D) Complete argument
(E) Complete argument
Goal: Find the argument that employs the
most similar argumentative strategy.
Parallel Argument questions
Conclusion
Premise
Premise
Premise
Assumption
Assumption+
(watch the entire video here)
Structure Questions
Question stem examples for Parallel Argument questions
Find the argument most like the original
• Which of the following arguments exhibits a pattern of
reasoning most similar to the pattern of reasoning
exhibited in the argument above?
• Which of the following is most like the argument above
in its logical structure?
• The pattern of reasoning displayed above is most
closely paralleled in which of the following?
(watch the entire video here)
Structure Questions
Strategy for tackling Parallel Argument questions
1. Identify and summarize the conclusion and premises
2. Use generic language to describe the method of reasoning
to yourself before checking the answer choices
3. Look for an argument with same structure
4. Check all answer choices
(watch the entire video here)
Structure Questions
Tips for Parallel Argument questions
1. Beware of answer choices with same subject matter
2. Questions are time-consuming check your time
(watch the entire video here)
Structure Questions
• Test your understanding of the argumentative strategies
employed in an argument
• 3 types of Structure Questions:
- Method of Reasoning
- Boldfaced
- Parallel Argument
(watch the entire video here)
Practice Question
ABC Widget’s plan to decrease employee salaries by 15% makes sense.
Granted, the reduced salaries will place a financial burden on many
employees. But, if the company does not decrease employee salaries, it
will not have enough money to stay in business, and everyone at the
company will lose his or her job.
The passage employs which of the following argumentative strategies?
A. It explains why the alternative course
of action would not be subject to the
objections raised against the
proposed course of action.
B. It describes an ideal situation by way
of a situation that is less than ideal.
C. It indirectly arrives at its conclusion
by providing grounds to reject an
alternative approach.
D. It extrapolates the perceived outcome of
the proposed plan in order to criticize the
alternative course of action.
E. It employs circular reasoning by assuming
that which it is trying to prove.
Practice Question
A. It explains why the alternative course
of action would not be subject to the
objections raised against the
proposed course of action.
B. It describes an ideal situation by way
of a situation that is less than ideal.
C. It indirectly arrives at its conclusion
by providing grounds to reject an
alternative approach.
D. It extrapolates the perceived outcome of
the proposed plan in order to criticize the
alternative course of action.
E. It employs circular reasoning by assuming
that which it is trying to prove.
P: 15% cut will hurt many
P: No cut  no jobs
C: 15% cut makes sense
Not doing X is much worse than
doing X. So, we should do X.




ABC Widget’s plan to decrease employee salaries by 15% makes sense.
Granted, the reduced salaries will place a financial burden on many
employees. But, if the company does not decrease employee salaries, it
will not have enough money to stay in business, and everyone at the
company will lose his or her job.
The passage employs which of the following argumentative strategies?
(watch the entire video here)
Practice Question
Fred: Columbus was a great explorer because he held on to his conviction that
the Earth was round in the face of overwhelming opposition.
Stan: The mark of a great explorer is bravery not the adherence to a
conviction. Besides, the concept of a round Earth was widely accepted
in the fifteenth century, when Columbus was looking for someone to
fund his voyage.
In Stan’s response, the two boldface portions play which of the following
roles?
A. The first presents an alternative
conclusion; the second provides evidence
in support of that conclusion.
B. The first rejects the criterion on which
Fred’s argument is based; the second
disputes a specific claim.
C. The first presents an alternative criterion;
the second describes a premise on which
Stan’s conclusion relies.
D. The first elaborates on Fred’s criterion;
the second presents a premise on which
Stan’s conclusion relies.
E. The first reveals a contradiction in Fred’s
argument; the second resolves that
contradiction.
Practice Question
Fred: Columbus was a great explorer because he held on to his conviction that
the Earth was round in the face of overwhelming opposition.
Stan: The mark of a great explorer is bravery not the adherence to a
conviction. Besides, the concept of a round Earth was widely accepted
in the fifteenth century, when Columbus was looking for someone to
fund his voyage.
In Stan’s response, the two boldface portions play which of the following
roles?
A. The first presents an alternative
conclusion; the second provides evidence
in support of that conclusion.
B. The first rejects the criterion on which
Fred’s argument is based; the second
disputes a specific claim.
C. The first presents an alternative criterion;
the second describes a premise on which
Stan’s conclusion relies.
D. The first elaborates on Fred’s criterion;
the second presents a premise on which
Stan’s conclusion relies.
E. The first reveals a contradiction in Fred’s
argument; the second resolves that
contradiction.
Fred
P: Conviction despite opposition
C: Columbus great explorer
Stan
1st: Great = bravery not conviction
2nd: Round earth known
Stan
1st: Refutes standard
2nd: Calls statement false




(watch the entire video here)
Practice Question
In Townville, most smokers play tennis, and most nonsmokers do not play tennis.
Therefore, in Townville, most tennis players smoke.
Which of the following exhibits a pattern of flawed reasoning most similar to that in
the argument above?
A. In Townville, most Lions Club members were born in
Townville, and most of the residents who are not Lions
Club members were not born in Townville. Therefore,
most of the residents who were born in Townville are
Lions Club members.
B. In Townville, most of the people who live west of Main
Street own a GPS, and most of the people who own a
GPS live east of Main street. Therefore, most of the
people in Townville own a GPS.
C. In Townville, most cat owners own exactly one dog, and
most dog owners own more than one dog. Therefore,
most of the people in Townville who own more than one
dog do not own any cats.
D. In Townville, most tennis players play golf, but not every
golfer plays tennis. Therefore, in Townville, there are
more tennis players than golfers.
E. In Townville, most of the houses are painted red, and
most of the houses have a pool. Therefore, in Townville,
most of the houses are painted red and have a pool.
Practice Question
In Townville, most smokers play tennis, and most nonsmokers do not play tennis.
Therefore, in Townville, most tennis players smoke.
Which of the following exhibits a pattern of flawed reasoning most similar to that in
the argument above?
P: S T
P: ~S ~T
C: T S
A. In Townville, most Lions Club members were born in
Townville, and most of the residents who are not Lions
Club members were not born in Townville. Therefore,
most of the residents who were born in Townville are
Lions Club members.
B. In Townville, most of the people who live west of Main
Street own a GPS, and most of the people who own a
GPS live east of Main street. Therefore, most of the
people in Townville own a GPS.
C. In Townville, most cat owners own exactly one dog, and
most dog owners own more than one dog. Therefore,
most of the people in Townville who own more than one
dog do not own any cats.
D. In Townville, most tennis players play golf, but not every
golfer plays tennis. Therefore, in Townville, there are
more tennis players than golfers.
E. In Townville, most of the houses are painted red, and
most of the houses have a pool. Therefore, in Townville,
most of the houses are painted red and have a pool.




(watch the entire video here)
Flawed Argument Questions (watch the entire video here)
(A) Main problem
(B) Main problem
(C) Main problem
(D) Main problem
(E) Main problem
Goal: Find the argument’s primary flaw
Flawed Conclusion
Premise
Premise
Premise
Assumption
Assumption+
Flawed Argument Questions (watch the entire video here)
Question stem examples
Identify the main problem with the argument
Flawed Argument Questions
• Which of the following identifies the most serious logical flaw
in the argument above?
• Which one of the following best identifies the error in
reasoning made in the passage?
• The argument is vulnerable to criticism on which one of the
following grounds?
• The reasoning in the argument is not sound because it fails to
establish that ___
(watch the entire video here)
1. Identify and summarize the conclusion and premises
2. Identify any unstated assumptions
3. Determine the primary flaw
4. Look for your answer among the answer choices
5. Check all answer choices
Flawed Argument Questions
Strategy
(watch the entire video here)
Common Flaws
Flawed Argument Questions
While studying the reading levels of elementary
students, researchers discovered that, on
average, the longer a child’s foot, the higher
his/her reading level. Therefore, foot growth
increases one’s reading level.
• Confusing causation with correlation
(watch the entire video here)
Common Flaws
• Confusing causation with correlation
• Confusing numbers with rates
Flawed Argument Questions
Last year, 10 people were murdered in
Happyton, and 100 people were murdered
in Killington. Therefore, it is much safer to
live in Happyton than in Killington
(watch the entire video here)
Common Flaws
• Confusing causation with correlation
• Confusing numbers with rates
• Conclusion mismatch
Flawed Argument Questions
Last year, hundreds of postal workers in Maltania
suffered back injuries from carrying heavy
packages at the sorting stations. To help reduce
these injuries, the government should introduce a
law that prohibits people from mailing packages
that are over 3 feet in length.
- Watch out for new words in the conclusion
(watch the entire video here)
Common Flaws
• Confusing causation with correlation
• Confusing numbers with rates
• Conclusion mismatch
Flawed Argument Questions
• Extreme conclusion
- Watch out for new words in the conclusion
In some countries, raising the minimum
wage can help boost the economy. So, if
Maltania raises its minimum wage, its
economy will improve.
(watch the entire video here)
Common Flaws
• Confusing causation with correlation
• Confusing numbers with rates
• Conclusion mismatch
Flawed Argument Questions
• Extreme conclusion
• Mistaking necessary for sufficient
- Watch out for new words in the conclusion
In order to be a great science teacher, one
must have a deep understanding of biology.
Since Terrence has a deep understanding of
biology, he must be a great science teacher.
(watch the entire video here)
Common Flaws
• Confusing causation with correlation
• Confusing numbers with rates
• Conclusion mismatch
Flawed Argument Questions
• Extreme conclusion
• Mistaking necessary for sufficient
• Guilty by association
- Watch out for new words in the conclusion
Sharon, Margaret and Rena are tall, Hoopton
High students who play basketball. Since
Maureen is a tall, Hoopton High student, she
must play basketball.
(watch the entire video here)
Common Flaws
• Confusing causation with correlation
• Confusing numbers with rates
• Conclusion mismatch
Flawed Argument Questions
• Extreme conclusion
• Mistaking necessary for sufficient
• Guilty by association
• Unrepresentative sample
- Watch out for new words in the conclusion
Recently, a questionnaire was given to the
inhabitants of Capton, Maltania’s capital city.
83% of respondents said they own one or more
horses. Therefore, we can conclude that most
people in Maltania own one or more horses.
(watch the entire video here)
Flawed Argument Questions
1. Identify and summarize the conclusion and premises
2. Identify any unstated assumptions
3. Determine the primary flaw
4. Look for your description among the answer choices
5. Check all answer choices
Strategy
(watch the entire video here)
Practice Question
A. The author assumes that the only
varieties of jabberwocky are
longhaired and shorthaired.
B. The author does not consider what
proportion of all cases of jimmylegs
involves longhaired jabberwockies.
C. The author calls into question the
authenticity of the WJS data without
providing any data to the contrary.
D. The author takes no account of the relative
frequency of longhaired jabberwockies
within the jabberwocky population.
E. The author limits the argument to an
unnecessarily restrictive subset of criteria
before drawing a general conclusion.
Jimmylegs, a disorder of the metatarsals, affects only jabberwockies. A popular
belief contends that the longhaired variety of the jabberwocky is more
predisposed to contract jimmylegs than are other varieties of jabberwocky. This,
however, is clearly a myth, since data collected by the World Jabberwocky
Society (WJS) indicate that, of all the jabberwockies that contract jimmylegs
each year, the majority of the cases involve shorthaired jabberwockies.
Which of the following highlights the most serious flaw in the above argument?
Practice Question
A. The author assumes that the only
varieties of jabberwocky are
longhaired and shorthaired.
B. The author does not consider what
proportion of all cases of jimmylegs
involves longhaired jabberwockies.
C. The author calls into question the
authenticity of the WJS data without
providing any data to the contrary.
D. The author takes no account of the relative
frequency of longhaired jabberwockies
within the jabberwocky population.
E. The author limits the argument to an
unnecessarily restrictive subset of criteria
before drawing a general conclusion.
Jimmylegs, a disorder of the metatarsals, affects only jabberwockies. A popular
belief contends that the longhaired variety of the jabberwocky is more
predisposed to contract jimmylegs than are other varieties of jabberwocky. This,
however, is clearly a myth, since data collected by the World Jabberwocky
Society (WJS) indicate that, of all the jabberwockies that contract jimmylegs
each year, the majority of the cases involve shorthaired jabberwockies.
Which of the following highlights the most serious flaw in the above argument?
A: Pop. breakdown supports conclusion
Flaw: We don’t know the
population breakdown




P: J-legs = jabberwocky disorder
P: Belief: longhair predisposed to j-legs
P: Majority of cases are shorthair
C: Longhair not more predisposed
(watch the entire video here)
Paradox Questions (watch the entire video here)
Goal: Find premise that resolves the paradox
Paradox Questions
Conclusion
Premise
Premise
Premise+
(A) New Premise
(B) New Premise
(C) New Premise
(D) New Premise
(E) New Premise
(watch the entire video here)
Question stem examples
Identify something that resolves the
contradictory information
Paradox Questions
• The paradox described above is best resolved by which of
the following?
• Which of the following, if true, most helps to resolve the
apparent discrepancy described above?
• Which of the following, if true, best explains the
paradoxical outcome of Dr. Doolittle's experiment?
• Which one of the following most helps to explain the
apparent contradiction above?
• Which one of the following, if true, most helps to explain
the difference in melting points?
(watch the entire video here)
Paradox Questions
Statistics show that the number of smokers in
Maltania has steadily decreased over the past 10
years. However, during the same 10 years, the
total amount of tobacco sold by Maltanian tobacco
farmers has increased.
Which of the following, if true, most helps to
resolve the apparent discrepancy described
above?
• Looking for an “aha” premise
• Not testing ability to deconstruct arguments
• Several explanations:
- Farmers exporting to other markets
- Big increase in tobacco chewers
- and more . . .
(watch the entire video here)
Does this explain why...?
Paradox Question strategy
1. Identify the contradictory premises
2. Explain the paradox to yourself
3. Check the answer choices while reminding yourself
of the paradox
4. Check all answer choices
Paradox Questions
e.g., More tobacco sold despite fewer smokers
(watch the entire video here)
Tips
• Keywords: yet, however, surprisingly, nonetheless, paradoxically
• Unable to identify paradox
Paradox Questions
solution unlikely
reread passage or guess and move on
• Beware of answer choices that have opposite effect
Statistics show that the number of smokers in Maltania
has steadily decreased over the past 10 years. However,
during the same 10 years, the total amount of tobacco
sold by Maltanian tobacco farmers has increased.
Which of the following, if true, most helps to resolve the
apparent discrepancy described above?
(A) The Maltanian government has introduced stop-smoking
programs across the country.

(watch the entire video here)
Practice Question
Among the mechanics at Joe’s Garage, the senior mechanics are the most
adept at diagnosing engine problems. However, the itemized bills for engine
repairs made during the past four years suggest otherwise. From the bills, we
see that, on average, the senior mechanics took 29 minutes to correctly
diagnose an engine problem, while the junior mechanics took only 22 minutes.
Which of the following, if true, most helps to resolve the apparent paradox?
A. At Joe’s Garage, mechanics with more
than three years of experience are
called senior mechanics.
B. Some of the junior mechanics had
experience diagnosing engine problems
before they began working at Joe’s
Garage.
C. At Joe’s Garage, the manager typically
assigns to the junior mechanics engine
problems that he expects will be
relatively easy to diagnose.
D. During the past 4 years, most of the
senior mechanics left Joe’s Garage to
open their own garages.
E. Mechanics who are more adept at
diagnosing engine problems are faster
at completing the required repair.
Practice Question
Among the mechanics at Joe’s Garage, the senior mechanics are the most
adept at diagnosing engine problems. However, the itemized bills for engine
repairs made during the past four years suggest otherwise. From the bills, we
see that, on average, the senior mechanics took 29 minutes to correctly
diagnose an engine problem, while the junior mechanics took only 22 minutes.
Which of the following, if true, most helps to resolve the apparent paradox?
P: Sr. mechs best at diagnosing
P: 29 min for sr. mechs, and
.22 min for jr. mechs
Paradox: Sr. mechanics take
longer despite their superior
diagnostic skills.
A. At Joe’s Garage, mechanics with more
than three years of experience are
called senior mechanics.
B. Some of the junior mechanics had
experience diagnosing engine problems
before they began working at Joe’s
Garage.
C. At Joe’s Garage, the manager typically
assigns to the junior mechanics engine
problems that he expects will be
relatively easy to diagnose.
D. During the past 4 years, most of the
senior mechanics left Joe’s Garage to
open their own garages.
E. Mechanics who are more adept at
diagnosing engine problems are faster
at completing the required repair.




(watch the entire video here)
Evaluate the Conclusion Questions (watch the entire video here)
(A) Question
(B) Question
(C) Question
(D) Question
(E) Question
Conclusion
Premise
Premise
Premise
Assumption
Assumption+
Evaluate the Conclusion Questions
Goal: Find the question that, when answered,
best helps to evaluate the conclusion.
(watch the entire video here)
Researcher: Two years ago, a wolf pack was relocated to Bilford
Island. Although the local rabbit population has decreased
drastically since the relocation, the wolves are not to blame for
this decrease. Our study shows that the unprecedented number
of recent rabbit deaths is due to the myxoma virus.
Which of the following would be most relevant to investigate in
order to evaluate the researcher’s conclusion?
Evaluate the Conclusion Questions
Conclusion: Wolves not responsible for population decrease.
A) In what season was the wolf pack relocated to Bilford Island?
B) What proportion of rabbits have died from the myxoma virus?
C) Did the wolf pack introduce the myxoma virus to Bilford Island?
D) . . .
Goal: Find the question that, when answered,
helps evaluate the conclusion
(watch the entire video here)
Question stem examples
Identify a question that would help
gauge the strength of the conclusion
Evaluate the Conclusion Questions
• Knowing which of the following would be most useful in evaluating
the argument?
• Which of the following would be most relevant to investigate in
order to evaluate the researcher’s conclusion?
• Clarification of which of the following issues would be most
important to evaluating the spokesperson’s position?
(watch the entire video here)
Strategy
1. Identify and summarize the conclusion and premises
2. Identify any assumptions
3. Check the answer choices by providing an answer to
each question and relating it to the conclusion
4. Check all answer choices
Evaluate the Conclusion Questions (watch the entire video here)
Evaluate the Conclusion Questions
Goal: Find the question that, when answered,
helps evaluate the conclusion.
3. Check the answer choices by providing an answer to
each question and relating it to the conclusion
(watch the entire video here)
Evaluate the Conclusion Questions
Conclusion: Wolves not responsible for population decrease.
3. Check the answer choices by providing an answer to
each question and relating it to the conclusion
A) In what season was the wolf pack relocated to Bilford Island?
Researcher: Two years ago, a wolf pack was relocated to Bilford
Island. Although the local rabbit population has decreased
drastically since the relocation, the wolves are not to blame for
this decrease. Our study shows that the unprecedented number
of recent rabbit deaths is due to the myxoma virus.
Which of the following would be most relevant to investigate to
evaluate the researcher’s conclusion?
Summer
(watch the entire video here)
Evaluate the Conclusion Questions
Conclusion: Wolves not responsible for population decrease.
3. Check the answer choices by providing an answer to
each question and relating it to the conclusion
A) In what season was the wolf pack relocated to Bilford Island?
Researcher: Two years ago, a wolf pack was relocated to Bilford
Island. Although the local rabbit population has decreased
drastically since the relocation, the wolves are not to blame for
this decrease. Our study shows that the unprecedented number
of recent rabbit deaths is due to the myxoma virus.
Which of the following would be most relevant to investigate to
evaluate the researcher’s conclusion?
Winter
(watch the entire video here)
Evaluate the Conclusion Questions
Conclusion: Wolves not responsible for population decrease.
3. Check the answer choices by providing an answer to
each question and relating it to the conclusion
A) In what season was the wolf pack relocated to Bilford Island?
B) What proportion of rabbits have died from the myxoma virus?
Researcher: Two years ago, a wolf pack was relocated to Bilford
Island. Although the local rabbit population has decreased
drastically since the relocation, the wolves are not to blame for
this decrease. Our study shows that the unprecedented number
of recent rabbit deaths is due to the myxoma virus.
Which of the following would be most relevant to investigate to
evaluate the researcher’s conclusion?
43%


Winter
(watch the entire video here)
Evaluate the Conclusion Questions
Conclusion: Wolves not responsible for population decrease.
3. Check the answer choices by providing an answer to
each question and relating it to the conclusion
A) In what season was the wolf pack relocated to Bilford Island?
B) What proportion of rabbits have died from the myxoma virus?
C) Did the wolf pack introduce the myxoma virus to Bilford Island?
Researcher: Two years ago, a wolf pack was relocated to Bilford
Island. Although the local rabbit population has decreased
drastically since the relocation, the wolves are not to blame for
this decrease. Our study shows that the unprecedented number
of recent rabbit deaths is due to the myxoma virus.
Which of the following would be most relevant to investigate to
evaluate the researcher’s conclusion?
43%



Winter
Yes
(watch the entire video here)
Evaluate the Conclusion Questions
Conclusion: Wolves not responsible for population decrease.
3. Check the answer choices by providing an answer to
each question and relating it to the conclusion
A) In what season was the wolf pack relocated to Bilford Island?
B) What proportion of rabbits have died from the myxoma virus?
C) Did the wolf pack introduce the myxoma virus to Bilford Island?
Researcher: Two years ago, a wolf pack was relocated to Bilford
Island. Although the local rabbit population has decreased
drastically since the relocation, the wolves are not to blame for
this decrease. Our study shows that the unprecedented number
of recent rabbit deaths is due to the myxoma virus.
Which of the following would be most relevant to investigate to
evaluate the researcher’s conclusion?
43%



Winter
No
(watch the entire video here)
Evaluate the Conclusion Questions
Strategy
1. Identify and summarize the conclusion and premises
2. Identify any assumptions
3. Check the answer choices by providing an answer to
each question and relating it to the conclusion
4. Check all answer choices
(watch the entire video here)
Practice Question
Professor Gelding: The belief that our planet will experience a wave of extreme
weather events over the next 100 years is nonsense. There is no evidence to
support that the rate of extreme weather events is increasing.
Professor Brink: To see where you are mistaken, you need only examine the
data concerning Category 5 hurricanes in the Gulf of Mexico that made landfall
in the past 100 years. In the first 50 years, there were only 6 such hurricanes,
but in the last 50 years, there were 19 such hurricanes.
Clarification of which of the following issues would be most important to an
evaluation of Professor Brink’s conclusion?
A. Does the rate at which Category 5 hurricanes
make landfall affect the rate of other extreme
weather events?
B. Do the statistics concerning Category 5
hurricanes that made landfall represent
extreme weather events in the Gulf of Mexico?
C. During the last 100 years, what percent of
hurricanes made landfall?
D. How many extreme weather events occurred in
the past 100 years?
E. Do the Category 5 hurricanes in the Gulf of
Mexico that made landfall in the past 100 years
represent extreme weather events in general?
Practice Question
A. Does the rate at which Category 5 hurricanes
make landfall affect the rate of other extreme
weather events?
B. Do the statistics concerning Category 5
hurricanes that made landfall represent
extreme weather events in the Gulf of Mexico?
C. During the last 100 years, what percent of
hurricanes made landfall?
D. How many extreme weather events occurred in
the past 100 years?
E. Do the Category 5 hurricanes in the Gulf of
Mexico that made landfall in the past 100 years
represent extreme weather events in general?
A: hurricane rate
characterizes extreme
weather rate
P: 1st 50 yrs: 6 hurricanes
P: 2nd 50 yrs: 19 hurricanes
C: EW rate is increasing
Professor Gelding: The belief that our planet will experience a wave of extreme
weather events over the next 100 years is nonsense. There is no evidence to
support that the rate of extreme weather events is increasing.
Professor Brink: To see where you are mistaken, you need only examine the
data concerning Category 5 hurricanes in the Gulf of Mexico that made landfall
in the past 100 years. In the first 50 years, there were only 6 such hurricanes,
but in the last 50 years, there were 19 such hurricanes.
Clarification of which of the following issues would be most important to an
evaluation of Professor Brink’s conclusion?




(watch the entire video here)
Miscellaneous Tips (watch the entire video here)
Miscellaneous Tips
• Question type frequencies
• GMAT words
• EXCEPT questions
• Being aggressive
(watch the entire video here)
Question type frequencies
Miscellaneous Tips
1. Weaken the Argument
2. Strengthen the Argument
3. Assumption
4. Conclusion/Inference
5. Method of Reasoning
6. Flawed Argument
7. Evaluation
8. Paradox
(watch the entire video here)
Question type frequencies
Miscellaneous Tips
1. Weaken the Argument
2. Strengthen the Argument
3. Assumption
4. Conclusion/Inference
5. Method of Reasoning
6. Flawed Argument
7. Evaluation
8. Paradox
¾+







(watch the entire video here)
Miscellaneous Tips
GMAT words
Everybody likes ice cream= a lot of people like ice cream
= most people like ice cream
Common usage
Everybody likes ice cream= every person likes ice cream
GMAT usage
(watch the entire video here)
Miscellaneous Tips
GMAT words
• Read words in their strongest, most literal sense
all, none, everyone, no one, always, never, each
every, anywhere, nowhere
• Some: 1 or more
Some Gigacorp employees are college graduates.
Some countries in Europe are named Italy.
Some of Earth’s oceans contain salt water



• Most: More than 50%
Most of Earth’s oceans contain salt water 
(watch the entire video here)
Miscellaneous Tips
EXCEPT questions
Today, Gary did not arrive at the office until 11am.
Therefore Gary will be fired.
Each of the following, if true, weakens the
conclusion above EXCEPT
• Does not mean “strengthen” the conclusion
(watch the entire video here)
Miscellaneous Tips
EXCEPT questions
Each of the following people is a former President
of the United States EXCEPT





If the person is a former US president
eliminate it.
(A) Abraham Lincoln
(B) George Washington
(C) Bugs Bunny
(D) Jimmy Carter
(E) George Bush
(watch the entire video here)
Miscellaneous Tips
EXCEPT questions
Today, Gary did not arrive at the office until 11am.
Therefore Gary will be fired.
Each of the following, if true, weakens the
conclusion above EXCEPT
If it weakens the conclusion, eliminate it.
(A) Work begins at noon
(B) Gary is self employed
(C) Gary likes ice cream



(watch the entire video here)
Miscellaneous Tips
Being aggressive
• Critical reasoning questions can be time consuming
• Students get lost in the answer choices
• Be aggressive!
“What if I eliminate all answer choices?”
• This will not happen often
• You will have earned time to recheck answer choices
Look for reasons to reject answer choices
(watch the entire video here)
Miscellaneous Tips
• Question type frequencies
• GMAT words
• EXCEPT questions
• Being aggressive
(watch the entire video here)
Practice Question
For decades, countries have measured wheat field productivity using the
Wheat Field Productivity Index (WFPI), which is equal to the average
number of pounds of grain harvested per acre of wheat. In 2005,
Maltania’s WFPI was 60 pounds per acre more than Italy’s WFPI. In 2006,
Maltania’s WFPI was 75 pounds per acre more than Italy’s. Therefore,
Maltania’s wheat field productivity must have increased during that period.
Which of the following, if true, provides the strongest support for the
conclusion above?
A. Between 2005 and 2006, the number of
acres of wheat planted in both Maltania
and Italy increased at the same rate.
B. In 2006, Maltania received 20% more
sunshine than usual.
C. Between 2005 and 2006, Italy’s wheat
field productivity decreased by 10
pounds per acre.
D. In 2006, Italy experienced its worst
drought in decades.
E. In 2006, Maltania began subsidizing all of
its wheat farmers.
Practice Question
P: WFPI = ave lbs wheat/acre
P: 2005: Maltania’s WFPI 60
more than Italy’s
P: 2006: Maltania’s WFPI 75
more than Italy’s
C: Maltania’s WFPI 
A. Between 2005 and 2006, the number of
acres of wheat planted in both Maltania
and Italy increased at the same rate.
B. In 2006, Maltania received 20% more
sunshine than usual.
C. Between 2005 and 2006, Italy’s wheat
field productivity decreased by 10
pounds per acre.
D. In 2006, Italy experienced its worst
drought in decades.
E. In 2006, Maltania began subsidizing all of
its wheat farmers.
For decades, countries have measured wheat field productivity using the
Wheat Field Productivity Index (WFPI), which is equal to the average
number of pounds of grain harvested per acre of wheat. In 2005,
Maltania’s WFPI was 60 pounds per acre more than Italy’s WFPI. In 2006,
Maltania’s WFPI was 75 pounds per acre more than Italy’s. Therefore,
Maltania’s wheat field productivity must have increased during that period.
Which of the following, if true, provides the strongest support for the
conclusion above?




(watch the entire video here)
Practice Question
All of Pedro’s friends say they know someone who is a licensed
circus clown. Pedro does not know any licensed circus clowns, and
he is not unique among his friends in this respect.
If the above statements are true, which of the following must also
be true?
A. Some of Pedro’s friends are lying.
B. Most licensed circus clowns lie about
their profession.
C. Some of Pedro’s friends have friends
that Pedro is not friends with.
D. Pedro is a licensed circus clown.
E. All of Pedro’s friends know the same
licensed circus clown.
Practice Question
All of Pedro’s friends say they know someone who is a licensed
circus clown. Pedro does not know any licensed circus clowns, and
he is not unique among his friends in this respect.
If the above statements are true, which of the following must also
be true?
A. Some of Pedro’s friends are lying.
B. Most licensed circus clowns lie about
their profession.
C. Some of Pedro’s friends have friends
that Pedro is not friends with.
D. Pedro is a licensed circus clown.
E. All of Pedro’s friends know the same
licensed circus clown.
P: All friends SAY they know a C
P: Pedro knows zero Cs
P: Some friends don’t know a C



(watch the entire video here)
Practice Question
Reporter: The number of foreigners working at US ski resorts was lower
last year than it had been in previous years. Last year, several ski
resorts hired more local residents than they had hired in previous years,
so it is likely that the reduced number of foreign workers at US ski
resorts last year was due to the decreased demand for their services.
Which of the following, if true, most seriously undermines the reporter’s
argument?
A. All foreigners who received work visas
for last year’s ski season were able to
find work at US ski resorts.
B. The average wait time to receive a US
work visa has decreased substantially
over the past three years.
C. The number of ski resorts that hired
foreigners last year was the same as in
previous years.
D. Foreigners can work for more than one
ski resort in any given year.
E. Recent changes to US immigration laws
drastically decreased the total number
of work visas issued for last year.
Practice Question
Reporter: The number of foreigners working at US ski resorts was lower
last year than it had been in previous years. Last year, several ski
resorts hired more local residents than they had hired in previous years,
so it is likely that the reduced number of foreign workers at US ski
resorts last year was due to the decreased demand for their services.
Which of the following, if true, most seriously undermines the reporter’s
argument?
A. All foreigners who received work visas
for last year’s ski season were able to
find work at US ski resorts.
B. The average wait time to receive a US
work visa has decreased substantially
over the past three years.
C. The number of ski resorts that hired
foreigners last year was the same as in
previous years.
D. Foreigners can work for more than one
ski resort in any given year.
E. Recent changes to US immigration laws
drastically decreased the total number
of work visas issued for last year.
A: Nothing else caused decrease
A: Fewer foreign workers did not
cause increase in locals hired




P: # foreign wrkrs lower last yr
P: More locals hired last yr
C:  local employees caused
 foreign employees
(watch the entire video here)
Practice Question
After a plague of locusts attacked every crop in Baker County, Kevin
Kevinson claimed that the severity of damage to his crops was due to
the fish-based fertilizer he applied to his fields before planting.
Which of the following investigations is most likely to yield significant
information that would help evaluate Kevin’s argument?
A. Comparing the value of Kevin’s crop damage
to the average value of the crop damage at
farms where fish-based fertilizer was not
applied to the fields before planting
B. Determining the extent to which fish-based
fertilizer increases crop yield
C. Comparing the long-term effects of Kevin’s
crop damage to the long-term effects of
other crop damage in Baker County
D. Comparing the severity of crop damage at Kevin’s
farm with the crop damage at agriculturally similar
Baker County farms where fish-based fertilizer was
not applied to fields before planting
E. Determining the percentage of Baker County farms
that applied fish-based fertilizer to their fields
before planting
A. Comparing the value of Kevin’s crop damage
to the average value of the crop damage at
farms where fish-based fertilizer was not
applied to the fields before planting
B. Determining the extent to which fish-based
fertilizer increases crop yield
C. Comparing the long-term effects of Kevin’s
crop damage to the long-term effects of
other crop damage in Baker County
D. Comparing the severity of crop damage at Kevin’s
farm with the crop damage at agriculturally similar
Baker County farms where fish-based fertilizer was
not applied to fields before planting
E. Determining the percentage of Baker County farms
that applied fish-based fertilizer to their fields
before planting
Practice Question
After a plague of locusts attacked every crop in Baker County, Kevin
Kevinson claimed that the severity of damage to his crops was due to
the fish-based fertilizer he applied to his fields before planting.
Which of the following investigations is most likely to yield significant
information that would help evaluate Kevin’s argument?
P: Locusts hit all crops
P: Kevin applied FBF to crops
C: FBF caused severity
A: Nothing else caused the
severity




(watch the entire video here)
Practice Question
Ayla is taller than Elan, and Brek is the same height as Diego. Since
Chayna is shorter than Diego, it follows that Ayla is taller than Chayna.
Each of the following, when added to the argument as an additional
premise, makes the argument logically correct EXCEPT:
A. Diego is shorter than Elan.
B. Ayla and Diego are the same height.
C. Brek is taller than Elan.
D. Elan and Chayna are the same height.
E. Brek is shorter than Ayla.
Practice Question
A. Diego is shorter than Elan.
B. Ayla and Diego are the same height.
C. Brek is taller than Elan.
D. Elan and Chayna are the same height.
E. Brek is shorter than Ayla.
If it makes the argument
correct, eliminate it.
A
C
Premises:
Conclusion:
+
B=D
C
A
E




Ayla is taller than Elan, and Brek is the same height as Diego. Since
Chayna is shorter than Diego, it follows that Ayla is taller than Chayna.
Each of the following, when added to the argument as an additional
premise, makes the argument logically correct EXCEPT:
(watch the entire video here)
Practice Question
A. In the past 7 years, the CPT for king crabs
has decreased by 50%.
B. 8 years ago, commercial crab-fishing
boats began using high-tech sonar
equipment that enables them to locate
crabs with much greater accuracy.
C. 9 years ago, a deadly species of sea lice
drove many crab species to near
extinction.
D. In the past 10 years, the number of
commercial crab traps set each year has
remained relatively constant.
E. In the past 10 years, the worldwide price
of snow crabs has more than doubled.
To estimate changes in crab populations, biologists monitor what is
known as “crabs per trap” (CPT). The CPT is the average number of
crabs that commercial crab fishermen catch per crab trap. Since the
current CPT for snow crabs is the same as it was 10 years ago,
biologists conclude that today’s snow crab population is approximately
the same as it was 10 years ago.
Which of the following, if true, most seriously weakens the argument?
Practice Question
A. In the past 7 years, the CPT for king crabs
has decreased by 50%.
B. 8 years ago, commercial crab-fishing
boats began using high-tech sonar
equipment that enables them to locate
crabs with much greater accuracy.
C. 9 years ago, a deadly species of sea lice
drove many crab species to near
extinction.
D. In the past 10 years, the number of
commercial crab traps set each year has
remained relatively constant.
E. In the past 10 years, the worldwide price
of snow crabs has more than doubled.
To estimate changes in crab populations, biologists monitor what is
known as “crabs per trap” (CPT). The CPT is the average number of
crabs that commercial crab fishermen catch per crab trap. Since the
current CPT for snow crabs is the same as it was 10 years ago,
biologists conclude that today’s snow crab population is approximately
the same as it was 10 years ago.
Which of the following, if true, most seriously weakens the argument?
P: CPT = ave # crabs/trap
P: CPT now = 10 yrs ago
C: SC pop. ≈ 10 yrs ago
A: CPT reflects pop.




(watch the entire video here)
Practice Question
Identical twins have livers that are genetically identical. Scientists recently
discovered that when one twin has hepachrinosis, a debilitating liver disease,
the caudate lobe of the affected twin’s liver is significantly smaller than the
caudate lobe of the unaffected twin’s liver. No such difference is found when
neither twin has hepachrinosis. From this information, it can be concluded
that hepachrinosis is caused by diminished liver capacity.
Which of the following is an assumption required by the argument?
A. Many diseases are caused by
diminished organ capacity.
B. The caudate lobe of a healthy identical
twin is the same size as the caudate
lobe of a healthy person who is not
an identical twin.
C. When two identical twins both suffer
from hepachrinosis, their caudate
lobes are the same size.
D. The relative smallness of the caudate lobe
of an affected person is not the result of
medications used in the treatment of
hepachrinosis.
E. People who have an identical twin are no
more likely to contract hepachrinosis than
people who do not have an identical twin.
Practice Question
Identical twins have livers that are genetically identical. Scientists recently
discovered that when one twin has hepachrinosis, a debilitating liver disease,
the caudate lobe of the affected twin’s liver is significantly smaller than the
caudate lobe of the unaffected twin’s liver. No such difference is found when
neither twin has hepachrinosis. From this information, it can be concluded
that hepachrinosis is caused by diminished liver capacity.
Which of the following is an assumption required by the argument?
A. Many diseases are caused by
diminished organ capacity.
B. The caudate lobe of a healthy identical
twin is the same size as the caudate
lobe of a healthy person who is not
an identical twin.
C. When two identical twins both suffer
from hepachrinosis, their caudate
lobes are the same size.
D. The relative smallness of the caudate lobe
of an affected person is not the result of
medications used in the treatment of
hepachrinosis.
E. People who have an identical twin are no
more likely to contract hepachrinosis than
people who do not have an identical twin.
P: Twins have = livers
P: Lobe of affected twin is smaller
P: Lobes of unaffected twins are =
C: Small lobe causes H
A: nothing else causes H




(watch the entire video here)
Practice Question
On average, when 100 non-pregnant women take the Pregunda home
pregnancy test, 10 of them will test positive. That is, the test will indicate
that the women are pregnant. Conversely, when 100 pregnant women
take the Pregunda home pregnancy test, 99 of them will test positive.
Therefore, if a group of women take the Pregunda home pregnancy test,
the vast majority of those who test positive will be pregnant.
The above argument is flawed primarily because it
A. does not take into consideration
the test’s margin of error
B. suggests that non-pregnant
women are more likely to test
positive than pregnant women are
C. presupposes the proportion of
pregnant women in the group
D. assumes that the test’s accuracy
is not affected by other factors
E. ignores the fact that women
who do not test positive may
be pregnant
Practice Question
A. does not take into consideration
the test’s margin of error
B. suggests that non-pregnant
women are more likely to test
positive than pregnant women are
C. presupposes the proportion of
pregnant women in the group
D. assumes that the test’s accuracy
is not affected by other factors
E. ignores the fact that women
who do not test positive may
be pregnant
On average, when 100 non-pregnant women take the Pregunda home
pregnancy test, 10 of them will test positive. That is, the test will indicate
that the women are pregnant. Conversely, when 100 pregnant women
take the Pregunda home pregnancy test, 99 of them will test positive.
Therefore, if a group of women take the Pregunda home pregnancy test,
the vast majority of those who test positive will be pregnant.
The above argument is flawed primarily because it
A: Test accuracy is constant




P: 10% of NP test positive
P: 99% of P test positive
C: Group  majority of
positive are pregnant
(watch the entire video here)
Practice Question
Jesse: The H1 vaccine prevents people from contracting thramboxia, a
disease that can cause short-term paralysis. While thramboxia is an
unpleasant disease with paralysis lasting up to 3 months, the H1
vaccine is not worth getting since hundreds of people die each year
from the H1 vaccination.
Harjit: But thousands of people die each year as a direct result of
contracting thramboxia. So, in addition to preventing thramboxia, the
H1 vaccine prevents deaths.
Harjit responds to Jesse by
A. questioning the truth of a statement on
which Jesse’s conclusion is based
B. suggesting that Jesse’s argument
overlooks an important consequence
C. noting that Jesse’s argument contradicts
that which he is trying to prove
D. reinforcing Jesse’s conclusion before
suggesting a new conclusion
E. citing evidence that disproves the
evidence cited by Jesse in drawing
his conclusion
Practice Question
Jesse: The H1 vaccine prevents people from contracting thramboxia, a
disease that can cause short-term paralysis. While thramboxia is an
unpleasant disease with paralysis lasting up to 3 months, the H1
vaccine is not worth getting since hundreds of people die each year
from the H1 vaccination.
Harjit: But thousands of people die each year as a direct result of
contracting thramboxia. So, in addition to preventing thramboxia, the
H1 vaccine prevents deaths.
Harjit responds to Jesse by
A. questioning the truth of a statement on
which Jesse’s conclusion is based
B. suggesting that Jesse’s argument
overlooks an important consequence
C. noting that Jesse’s argument contradicts
that which he is trying to prove
D. reinforcing Jesse’s conclusion before
suggesting a new conclusion
E. citing evidence that disproves the
evidence cited by Jesse in drawing
his conclusion
P: H1 prevents T
P: T can cause paralysis
P: 100s die from H1
C: Don’t get H1 vaccine
P: 1000’s die from T
P: H1 prevents T & deaths
C: Get H1 vaccine
There’s also Y. My option
accomplishes 2 things




(watch the entire video here)
GMAT Critical Reasoning - everything you need to know
GMAT Critical Reasoning - everything you need to know
GMAT Critical Reasoning - everything you need to know
GMAT Critical Reasoning - everything you need to know
GMAT Critical Reasoning - everything you need to know
GMAT Critical Reasoning - everything you need to know
GMAT Critical Reasoning - everything you need to know
GMAT Critical Reasoning - everything you need to know
GMAT Critical Reasoning - everything you need to know
GMAT Critical Reasoning - everything you need to know

More Related Content

What's hot

IELTS Writing Common Grammar Mistakes
IELTS Writing Common Grammar MistakesIELTS Writing Common Grammar Mistakes
IELTS Writing Common Grammar MistakesJROOZ Review Center
 
Comparatives and superlatives
Comparatives and superlativesComparatives and superlatives
Comparatives and superlativesnancyvega61
 
IELTS Task 2 Agree or Disagree Lesson
IELTS Task 2 Agree or Disagree Lesson IELTS Task 2 Agree or Disagree Lesson
IELTS Task 2 Agree or Disagree Lesson British Council
 
IELTS Reading Overview, Tips and Matching Headings
IELTS Reading Overview, Tips and Matching Headings IELTS Reading Overview, Tips and Matching Headings
IELTS Reading Overview, Tips and Matching Headings British Council
 
Academic reading (1)ielt presentation
Academic reading (1)ielt presentationAcademic reading (1)ielt presentation
Academic reading (1)ielt presentationAlexander Benito
 
IELTS Writing Task 2
IELTS Writing Task 2IELTS Writing Task 2
IELTS Writing Task 2Accent BLC
 
Complete Guide to Learning Verb Tenses.pdf
Complete Guide to Learning Verb Tenses.pdfComplete Guide to Learning Verb Tenses.pdf
Complete Guide to Learning Verb Tenses.pdfChloe Cheney
 
PARTS OFSPEECH POWERPOINT PRESENTATION ACTIVITY (1).pdf
PARTS OFSPEECH POWERPOINT PRESENTATION ACTIVITY (1).pdfPARTS OFSPEECH POWERPOINT PRESENTATION ACTIVITY (1).pdf
PARTS OFSPEECH POWERPOINT PRESENTATION ACTIVITY (1).pdfMaraRebecaAvendao
 
Vocabulary in context
Vocabulary in contextVocabulary in context
Vocabulary in contextehvienne01
 
ielts-writing-task-1-writing.pdf
ielts-writing-task-1-writing.pdfielts-writing-task-1-writing.pdf
ielts-writing-task-1-writing.pdfssuser85a8ea
 
IELTS Reading Preparation Tips
IELTS Reading Preparation TipsIELTS Reading Preparation Tips
IELTS Reading Preparation TipsJulia Robert
 
Ielts writing task 2
Ielts writing task 2Ielts writing task 2
Ielts writing task 2Aldyansyah -
 
IELTS Listening - MCQs -Intorduction - Useful Tips
IELTS Listening - MCQs -Intorduction - Useful TipsIELTS Listening - MCQs -Intorduction - Useful Tips
IELTS Listening - MCQs -Intorduction - Useful TipsIELTSBackup
 
Clause (Part-9 of 10)-Adverb Clause
Clause (Part-9 of 10)-Adverb ClauseClause (Part-9 of 10)-Adverb Clause
Clause (Part-9 of 10)-Adverb ClauseMd. Abdul Kader
 
Sentence transformation part 1
Sentence transformation part   1Sentence transformation part   1
Sentence transformation part 1Anil Kumar
 
Principal auxiliaries
Principal auxiliariesPrincipal auxiliaries
Principal auxiliariesmaahwash
 

What's hot (20)

IELTS Writing Common Grammar Mistakes
IELTS Writing Common Grammar MistakesIELTS Writing Common Grammar Mistakes
IELTS Writing Common Grammar Mistakes
 
Comparatives and superlatives
Comparatives and superlativesComparatives and superlatives
Comparatives and superlatives
 
IELTS Task 2 Agree or Disagree Lesson
IELTS Task 2 Agree or Disagree Lesson IELTS Task 2 Agree or Disagree Lesson
IELTS Task 2 Agree or Disagree Lesson
 
Antonyms
AntonymsAntonyms
Antonyms
 
IELTS Reading Overview, Tips and Matching Headings
IELTS Reading Overview, Tips and Matching Headings IELTS Reading Overview, Tips and Matching Headings
IELTS Reading Overview, Tips and Matching Headings
 
Academic reading (1)ielt presentation
Academic reading (1)ielt presentationAcademic reading (1)ielt presentation
Academic reading (1)ielt presentation
 
IELTS Writing Task 2
IELTS Writing Task 2IELTS Writing Task 2
IELTS Writing Task 2
 
Complete Guide to Learning Verb Tenses.pdf
Complete Guide to Learning Verb Tenses.pdfComplete Guide to Learning Verb Tenses.pdf
Complete Guide to Learning Verb Tenses.pdf
 
PARTS OFSPEECH POWERPOINT PRESENTATION ACTIVITY (1).pdf
PARTS OFSPEECH POWERPOINT PRESENTATION ACTIVITY (1).pdfPARTS OFSPEECH POWERPOINT PRESENTATION ACTIVITY (1).pdf
PARTS OFSPEECH POWERPOINT PRESENTATION ACTIVITY (1).pdf
 
Assessment report
Assessment reportAssessment report
Assessment report
 
Vocabulary in context
Vocabulary in contextVocabulary in context
Vocabulary in context
 
ielts-writing-task-1-writing.pdf
ielts-writing-task-1-writing.pdfielts-writing-task-1-writing.pdf
ielts-writing-task-1-writing.pdf
 
Adverb
AdverbAdverb
Adverb
 
Adjectives
AdjectivesAdjectives
Adjectives
 
IELTS Reading Preparation Tips
IELTS Reading Preparation TipsIELTS Reading Preparation Tips
IELTS Reading Preparation Tips
 
Ielts writing task 2
Ielts writing task 2Ielts writing task 2
Ielts writing task 2
 
IELTS Listening - MCQs -Intorduction - Useful Tips
IELTS Listening - MCQs -Intorduction - Useful TipsIELTS Listening - MCQs -Intorduction - Useful Tips
IELTS Listening - MCQs -Intorduction - Useful Tips
 
Clause (Part-9 of 10)-Adverb Clause
Clause (Part-9 of 10)-Adverb ClauseClause (Part-9 of 10)-Adverb Clause
Clause (Part-9 of 10)-Adverb Clause
 
Sentence transformation part 1
Sentence transformation part   1Sentence transformation part   1
Sentence transformation part 1
 
Principal auxiliaries
Principal auxiliariesPrincipal auxiliaries
Principal auxiliaries
 

Similar to GMAT Critical Reasoning - everything you need to know

Presentation of what i have learnt in film
Presentation of what i have learnt in filmPresentation of what i have learnt in film
Presentation of what i have learnt in filmseanholder95
 
Test video reflections proforma 2021
Test video reflections proforma 2021Test video reflections proforma 2021
Test video reflections proforma 2021ClaudiaRose5
 
Test Video Reflections Proforma 2021
Test Video Reflections Proforma 2021Test Video Reflections Proforma 2021
Test Video Reflections Proforma 2021AidanJones20
 
To Do List and Preliminary Task
To Do List and Preliminary TaskTo Do List and Preliminary Task
To Do List and Preliminary Taskowchsmedia
 
Unit 3 skills development task 3 luc john raubenheimer
Unit 3 skills development task 3 luc john raubenheimerUnit 3 skills development task 3 luc john raubenheimer
Unit 3 skills development task 3 luc john raubenheimerCeliaHuang5
 
Harry Statham Test video reflections proforma 2021
Harry Statham Test video reflections proforma 2021 Harry Statham Test video reflections proforma 2021
Harry Statham Test video reflections proforma 2021 HarryStatham
 

Similar to GMAT Critical Reasoning - everything you need to know (8)

Interactive Flashcards for GRE Math
Interactive Flashcards for GRE MathInteractive Flashcards for GRE Math
Interactive Flashcards for GRE Math
 
Presentation of what i have learnt in film
Presentation of what i have learnt in filmPresentation of what i have learnt in film
Presentation of what i have learnt in film
 
Test video reflections proforma 2021
Test video reflections proforma 2021Test video reflections proforma 2021
Test video reflections proforma 2021
 
Test Video Reflections Proforma 2021
Test Video Reflections Proforma 2021Test Video Reflections Proforma 2021
Test Video Reflections Proforma 2021
 
To Do List and Preliminary Task
To Do List and Preliminary TaskTo Do List and Preliminary Task
To Do List and Preliminary Task
 
2. Research MV
2. Research MV2. Research MV
2. Research MV
 
Unit 3 skills development task 3 luc john raubenheimer
Unit 3 skills development task 3 luc john raubenheimerUnit 3 skills development task 3 luc john raubenheimer
Unit 3 skills development task 3 luc john raubenheimer
 
Harry Statham Test video reflections proforma 2021
Harry Statham Test video reflections proforma 2021 Harry Statham Test video reflections proforma 2021
Harry Statham Test video reflections proforma 2021
 

Recently uploaded

The basics of sentences session 2pptx copy.pptx
The basics of sentences session 2pptx copy.pptxThe basics of sentences session 2pptx copy.pptx
The basics of sentences session 2pptx copy.pptxheathfieldcps1
 
microwave assisted reaction. General introduction
microwave assisted reaction. General introductionmicrowave assisted reaction. General introduction
microwave assisted reaction. General introductionMaksud Ahmed
 
The Most Excellent Way | 1 Corinthians 13
The Most Excellent Way | 1 Corinthians 13The Most Excellent Way | 1 Corinthians 13
The Most Excellent Way | 1 Corinthians 13Steve Thomason
 
Interactive Powerpoint_How to Master effective communication
Interactive Powerpoint_How to Master effective communicationInteractive Powerpoint_How to Master effective communication
Interactive Powerpoint_How to Master effective communicationnomboosow
 
Sanyam Choudhary Chemistry practical.pdf
Sanyam Choudhary Chemistry practical.pdfSanyam Choudhary Chemistry practical.pdf
Sanyam Choudhary Chemistry practical.pdfsanyamsingh5019
 
Incoming and Outgoing Shipments in 1 STEP Using Odoo 17
Incoming and Outgoing Shipments in 1 STEP Using Odoo 17Incoming and Outgoing Shipments in 1 STEP Using Odoo 17
Incoming and Outgoing Shipments in 1 STEP Using Odoo 17Celine George
 
Presentation by Andreas Schleicher Tackling the School Absenteeism Crisis 30 ...
Presentation by Andreas Schleicher Tackling the School Absenteeism Crisis 30 ...Presentation by Andreas Schleicher Tackling the School Absenteeism Crisis 30 ...
Presentation by Andreas Schleicher Tackling the School Absenteeism Crisis 30 ...EduSkills OECD
 
Science 7 - LAND and SEA BREEZE and its Characteristics
Science 7 - LAND and SEA BREEZE and its CharacteristicsScience 7 - LAND and SEA BREEZE and its Characteristics
Science 7 - LAND and SEA BREEZE and its CharacteristicsKarinaGenton
 
A Critique of the Proposed National Education Policy Reform
A Critique of the Proposed National Education Policy ReformA Critique of the Proposed National Education Policy Reform
A Critique of the Proposed National Education Policy ReformChameera Dedduwage
 
Introduction to ArtificiaI Intelligence in Higher Education
Introduction to ArtificiaI Intelligence in Higher EducationIntroduction to ArtificiaI Intelligence in Higher Education
Introduction to ArtificiaI Intelligence in Higher Educationpboyjonauth
 
Kisan Call Centre - To harness potential of ICT in Agriculture by answer farm...
Kisan Call Centre - To harness potential of ICT in Agriculture by answer farm...Kisan Call Centre - To harness potential of ICT in Agriculture by answer farm...
Kisan Call Centre - To harness potential of ICT in Agriculture by answer farm...Krashi Coaching
 
Crayon Activity Handout For the Crayon A
Crayon Activity Handout For the Crayon ACrayon Activity Handout For the Crayon A
Crayon Activity Handout For the Crayon AUnboundStockton
 
Employee wellbeing at the workplace.pptx
Employee wellbeing at the workplace.pptxEmployee wellbeing at the workplace.pptx
Employee wellbeing at the workplace.pptxNirmalaLoungPoorunde1
 
Software Engineering Methodologies (overview)
Software Engineering Methodologies (overview)Software Engineering Methodologies (overview)
Software Engineering Methodologies (overview)eniolaolutunde
 
Enzyme, Pharmaceutical Aids, Miscellaneous Last Part of Chapter no 5th.pdf
Enzyme, Pharmaceutical Aids, Miscellaneous Last Part of Chapter no 5th.pdfEnzyme, Pharmaceutical Aids, Miscellaneous Last Part of Chapter no 5th.pdf
Enzyme, Pharmaceutical Aids, Miscellaneous Last Part of Chapter no 5th.pdfSumit Tiwari
 
URLs and Routing in the Odoo 17 Website App
URLs and Routing in the Odoo 17 Website AppURLs and Routing in the Odoo 17 Website App
URLs and Routing in the Odoo 17 Website AppCeline George
 
BASLIQ CURRENT LOOKBOOK LOOKBOOK(1) (1).pdf
BASLIQ CURRENT LOOKBOOK  LOOKBOOK(1) (1).pdfBASLIQ CURRENT LOOKBOOK  LOOKBOOK(1) (1).pdf
BASLIQ CURRENT LOOKBOOK LOOKBOOK(1) (1).pdfSoniaTolstoy
 
How to Make a Pirate ship Primary Education.pptx
How to Make a Pirate ship Primary Education.pptxHow to Make a Pirate ship Primary Education.pptx
How to Make a Pirate ship Primary Education.pptxmanuelaromero2013
 

Recently uploaded (20)

The basics of sentences session 2pptx copy.pptx
The basics of sentences session 2pptx copy.pptxThe basics of sentences session 2pptx copy.pptx
The basics of sentences session 2pptx copy.pptx
 
microwave assisted reaction. General introduction
microwave assisted reaction. General introductionmicrowave assisted reaction. General introduction
microwave assisted reaction. General introduction
 
The Most Excellent Way | 1 Corinthians 13
The Most Excellent Way | 1 Corinthians 13The Most Excellent Way | 1 Corinthians 13
The Most Excellent Way | 1 Corinthians 13
 
Interactive Powerpoint_How to Master effective communication
Interactive Powerpoint_How to Master effective communicationInteractive Powerpoint_How to Master effective communication
Interactive Powerpoint_How to Master effective communication
 
Model Call Girl in Tilak Nagar Delhi reach out to us at 🔝9953056974🔝
Model Call Girl in Tilak Nagar Delhi reach out to us at 🔝9953056974🔝Model Call Girl in Tilak Nagar Delhi reach out to us at 🔝9953056974🔝
Model Call Girl in Tilak Nagar Delhi reach out to us at 🔝9953056974🔝
 
Sanyam Choudhary Chemistry practical.pdf
Sanyam Choudhary Chemistry practical.pdfSanyam Choudhary Chemistry practical.pdf
Sanyam Choudhary Chemistry practical.pdf
 
Incoming and Outgoing Shipments in 1 STEP Using Odoo 17
Incoming and Outgoing Shipments in 1 STEP Using Odoo 17Incoming and Outgoing Shipments in 1 STEP Using Odoo 17
Incoming and Outgoing Shipments in 1 STEP Using Odoo 17
 
Presentation by Andreas Schleicher Tackling the School Absenteeism Crisis 30 ...
Presentation by Andreas Schleicher Tackling the School Absenteeism Crisis 30 ...Presentation by Andreas Schleicher Tackling the School Absenteeism Crisis 30 ...
Presentation by Andreas Schleicher Tackling the School Absenteeism Crisis 30 ...
 
Código Creativo y Arte de Software | Unidad 1
Código Creativo y Arte de Software | Unidad 1Código Creativo y Arte de Software | Unidad 1
Código Creativo y Arte de Software | Unidad 1
 
Science 7 - LAND and SEA BREEZE and its Characteristics
Science 7 - LAND and SEA BREEZE and its CharacteristicsScience 7 - LAND and SEA BREEZE and its Characteristics
Science 7 - LAND and SEA BREEZE and its Characteristics
 
A Critique of the Proposed National Education Policy Reform
A Critique of the Proposed National Education Policy ReformA Critique of the Proposed National Education Policy Reform
A Critique of the Proposed National Education Policy Reform
 
Introduction to ArtificiaI Intelligence in Higher Education
Introduction to ArtificiaI Intelligence in Higher EducationIntroduction to ArtificiaI Intelligence in Higher Education
Introduction to ArtificiaI Intelligence in Higher Education
 
Kisan Call Centre - To harness potential of ICT in Agriculture by answer farm...
Kisan Call Centre - To harness potential of ICT in Agriculture by answer farm...Kisan Call Centre - To harness potential of ICT in Agriculture by answer farm...
Kisan Call Centre - To harness potential of ICT in Agriculture by answer farm...
 
Crayon Activity Handout For the Crayon A
Crayon Activity Handout For the Crayon ACrayon Activity Handout For the Crayon A
Crayon Activity Handout For the Crayon A
 
Employee wellbeing at the workplace.pptx
Employee wellbeing at the workplace.pptxEmployee wellbeing at the workplace.pptx
Employee wellbeing at the workplace.pptx
 
Software Engineering Methodologies (overview)
Software Engineering Methodologies (overview)Software Engineering Methodologies (overview)
Software Engineering Methodologies (overview)
 
Enzyme, Pharmaceutical Aids, Miscellaneous Last Part of Chapter no 5th.pdf
Enzyme, Pharmaceutical Aids, Miscellaneous Last Part of Chapter no 5th.pdfEnzyme, Pharmaceutical Aids, Miscellaneous Last Part of Chapter no 5th.pdf
Enzyme, Pharmaceutical Aids, Miscellaneous Last Part of Chapter no 5th.pdf
 
URLs and Routing in the Odoo 17 Website App
URLs and Routing in the Odoo 17 Website AppURLs and Routing in the Odoo 17 Website App
URLs and Routing in the Odoo 17 Website App
 
BASLIQ CURRENT LOOKBOOK LOOKBOOK(1) (1).pdf
BASLIQ CURRENT LOOKBOOK  LOOKBOOK(1) (1).pdfBASLIQ CURRENT LOOKBOOK  LOOKBOOK(1) (1).pdf
BASLIQ CURRENT LOOKBOOK LOOKBOOK(1) (1).pdf
 
How to Make a Pirate ship Primary Education.pptx
How to Make a Pirate ship Primary Education.pptxHow to Make a Pirate ship Primary Education.pptx
How to Make a Pirate ship Primary Education.pptx
 

GMAT Critical Reasoning - everything you need to know

  • 1. GMAT Critical Reasoning - Everything you need to know This slideshow features screenshots from GMAT Prep Now’s entire Critical Reasoning module (consisting of 38 videos). It covers everything you need to know, and it includes 24 practice questions. www.GMATPrepNow.com
  • 2. www.GMATPrepNow.com Note: since these slides are just snippets of a full-length video course, there may be times when you’re unable to glean all the relevant information from a particular screenshot. If, at any time, you’d like to watch the entire video on a certain topic, just click on the link at the top of that page, and you’ll be taken that that particular video. GMAT Critical Reasoning - Everything you need to know
  • 3. If you enjoy this learning format, let us know, and we’ll add similar resources to our SlideShare page GMAT Critical Reasoning - Everything you need to know
  • 4. Introduction to Critical Reasoning (watch the entire video here)
  • 5. This lesson features GMAT questions from the GMAT® mini-test. GMAT® questions are the property of the Graduate Management Admission Council® and are used here with their permission. Introduction to Critical Reasoning (watch the entire video here)
  • 6. • About 1/3 of Verbal questions (13 to 15 questions) • Batches of 2 or 3 questions • Approximately 2 minutes per question • Test your ability to reason effectively Introduction to Critical Reasoning (watch the entire video here)
  • 7. Introduction to Critical Reasoning The average normal infant born in the United States weighs between twelve and fourteen pounds at the age of three months. Therefore, if a three-month-old child weighs only ten pounds, its weight gain has been below the United States average. Which of the following indicates a flaw in the reasoning above? A) Weight is only one measure of normal infant development. B) Some three-month-old children weigh as much as seventeen pounds. C) It is possible for a normal child to weigh ten pounds at birth. D) The phrase "below average" does not necessarily mean insufficient. E) Average weight gain is not the same as average weight. Passage (watch the entire video here)
  • 8. Introduction to Critical Reasoning The average normal infant born in the United States weighs between twelve and fourteen pounds at the age of three months. Therefore, if a three-month-old child weighs only ten pounds, its weight gain has been below the United States average. Which of the following indicates a flaw in the reasoning above? A) Weight is only one measure of normal infant development. B) Some three-month-old children weigh as much as seventeen pounds. C) It is possible for a normal child to weigh ten pounds at birth. D) The phrase "below average" does not necessarily mean insufficient. E) Average weight gain is not the same as average weight. Passage Question stem (watch the entire video here)
  • 9. Introduction to Critical Reasoning The average normal infant born in the United States weighs between twelve and fourteen pounds at the age of three months. Therefore, if a three-month-old child weighs only ten pounds, its weight gain has been below the United States average. Which of the following indicates a flaw in the reasoning above? A) Weight is only one measure of normal infant development. B) Some three-month-old children weigh as much as seventeen pounds. C) It is possible for a normal child to weigh ten pounds at birth. D) The phrase "below average" does not necessarily mean insufficient. E) Average weight gain is not the same as average weight. Passage Question stem Answer choices (watch the entire video here)
  • 10. Introduction to Critical Reasoning • About 1/3 of Verbal questions (13 to 15 questions) • Batches of 2 or 3 questions • Approximately 2 minutes per question • Test your ability to reason effectively • Arguments consist of premises and a conclusion • Conclusion: what the author is trying to convince you of • Premises: the evidence used to support the conclusion (watch the entire video here)
  • 11. Introduction to Critical Reasoning Premise: All men are mortal Premise: Socrates is a man All men are mortal. Socrates is a man. Therefore, Socrates is mortal. Conclusion: Socrates is mortal + Premise: Gary late for last 3 days Premise: Flat tire this morning Conclusion: Gary will be late today + Strength of an Argument: How well the conclusion follows from the premises Inductive: the conclusion is not guaranteed Deductive: the conclusion is guaranteed For the past 3 days, Gary has arrived late for work. This morning, while Gary was driving to work, his car got a flat tire. Therefore, Gary will be late for work today. (watch the entire video here)
  • 12. Introduction to Critical Reasoning Strength of an Argument: How well the conclusion follows from the premises Strengthen Strengthen For the past 3 days, Gary has arrived late for work. This morning, while Gary was driving to work, his car got a flat tire. Therefore, Gary will be late for work today. For the past 53 days, Gary has arrived late for work. This morning, while Gary was driving to work, his car got a flat tire. Therefore, Gary will be late for work today. For the past 53 days, Gary has arrived late for work. This morning, while Gary was driving to work, his car got 4 flat tires, and his engine exploded. Therefore, Gary will be late for work today. (watch the entire video here)
  • 13. Introduction to Critical Reasoning Strength of an Argument: How well the conclusion follows from the premises Weaken For the past 3 days, Gary has arrived late for work. This morning, while Gary was driving to work, his car got a flat tire. Therefore, Gary will be late for work today. For the past 3 days, Gary has arrived late for work. This morning, while Gary was driving to work, his car got a flat tire. Gary’s house is 1 block from work. Therefore, Gary will be late for work today. (watch the entire video here)
  • 14. Introduction to Critical Reasoning Strength of an Argument: How well the conclusion follows from the premises • The conclusions in most arguments are not guaranteed • Most arguments can be strengthened or weakened by adding/altering premises For the past 3 days, Gary has arrived late for work. This morning, while Gary was driving to work, his car got a flat tire. Therefore, Gary will be late for work today. (watch the entire video here)
  • 15. Introduction to Critical Reasoning Strength of an Argument How well the conclusion follows from the premises Accept all premises as true! For the past 3 days, Gary has arrived late for work. This morning, while Gary was driving to work, his car got a flat tire. Therefore, Gary will be late for work today. (watch the entire video here)
  • 16. Introduction to Critical Reasoning All pigs can fly. Mount Everest is a pig. Therefore, Mount Everest can fly. Accept all premises as true!  (watch the entire video here)
  • 17. Introduction to Critical Reasoning All pigs can fly. Mount Everest is a pig. Therefore, Mount Everest cannot fly. Accept all premises as true!  (watch the entire video here)
  • 18. Introduction to Critical Reasoning • About 1/3 of Verbal questions (13 to 15 questions) • Batches of 2 or 3 questions • Approximately 2 minutes per question • Test your ability to reason effectively • Arguments consist of premises and a conclusion • Conclusion: what the author is trying to convince you of • Premises: the evidence used to support the conclusion • Assumption(s): unstated premise(s) necessary to reach conclusion (watch the entire video here)
  • 19. Introduction to Critical Reasoning Assumptions: • Gary’s history of lateness will be repeated today • The time needed to change tire will exceed the time before work starts • Gary has no other means by which to arrive at work on time For the past 3 days, Gary has arrived late for work. This morning, while Gary was driving to work, his car got a flat tire. Therefore, Gary will be late for work today. (watch the entire video here)
  • 20. Introduction to Critical Reasoning Premise(s) Conclusion +Assumption(s) (watch the entire video here)
  • 21. Introduction to Critical Reasoning • About 1/3 of Verbal questions (13 to 15 questions) • Batches of 2 or 3 questions • Approximately 2 minutes per question • Test your ability to reason effectively • Arguments consist of premises and a conclusion • Conclusion: what the author is trying to convince you of • Premises: the evidence used to support the conclusion • Assumption(s): unstated premise(s) necessary to reach conclusion Strength of an Argument: How well the conclusion follows from the premises Accept all premises as true! (watch the entire video here)
  • 22. Dissecting an Argument (watch the entire video here)
  • 23. Dissecting an Argument Every hockey fan I know is nice. I do not know Judy, but since she is wearing a hockey jersey, she must be nice. Summarize: - the conclusion - the premises - any assumptions (watch the entire video here)
  • 24. Dissecting an Argument “Is this what the author is trying to convince me of?” Every hockey fan I know is nice. I do not know Judy, but since she is wearing a hockey jersey, she must be nice. Conclusion: J is nice (watch the entire video here)
  • 25. Dissecting an Argument Every hockey fan I know is nice. I do not know Judy, but since she is wearing a hockey jersey, she must be nice. Goal: make your summaries as brief as possible (while maintaining coherence) Conclusion: J is nice (watch the entire video here)
  • 26. Dissecting an Argument Conclusion: J is nice Every hockey fan I know is nice. I do not know Judy, but since she is wearing a hockey jersey, she must be nice. Premise: All H-fans I know are nice Premise: I don’t know J Premise: J wearing jersey + Assumption: The H-fans I know are representative Assumption: Wearing jersey makes one a fan (watch the entire video here)
  • 27. Dissecting an Argument Researcher: Two years ago, a wolf pack was relocated to Bilford Island. Although the local rabbit population has decreased drastically since the relocation, the wolves are not to blame for this decrease. Our study shows that the unprecedented number of recent rabbit deaths is due to the myxoma virus. Summarize: - the conclusion - the premises - any assumptions (watch the entire video here)
  • 28. Dissecting an Argument Premise-Therefore-Conclusion test Researcher: Two years ago, a wolf pack was relocated to Bilford Island. Although the local rabbit population has decreased drastically since the relocation, the wolves are not to blame for this decrease. Our study shows that the unprecedented number of recent rabbit deaths is due to the myxoma virus. Option A: wolves not to blame therefore virus killed rabbits Option B: virus killed rabbits therefore wolves not to blame   (watch the entire video here)
  • 29. Dissecting an Argument Conclusion:  rabbit pop. not wolves’ fault + Researcher: Two years ago, a wolf pack was relocated to Bilford Island. Although the local rabbit population has decreased drastically since the relocation, the wolves are not to blame for this decrease. Our study shows that the unprecedented number of recent rabbit deaths is due to the myxoma virus. Premise: wolves arrived 2 yrs ago Premise: rabbit pop.  since Premise: virus caused deaths Assumption: wolves didn’t contribute to virus (watch the entire video here)
  • 30. Dissecting an Argument Tips for Identifying Conclusions and Premises • Watch for trigger words that indicate a conclusion • Watch for trigger words that indicate a premise therefore, thus, hence, so, implies, indicates, consequently, as a result, clearly, accordingly, infer, conclude since, because, for, due to, evidence, on the basis of, given that • Beware of common argument structures 1. Premise, premise, . . . , conclusion 2. Conclusion, premise, premise . . . 3. Conclusion in the question stem (watch the entire video here)
  • 31. Dissecting an Argument Summarize: - the conclusion - the premises - any assumptions Until recently, the only fish species living in Chilliwack Lake was the Gigafish. Last month, however, several Sovkafish were spotted in the lake. Unlike Gigafish, Sovkafish do not eat insects; instead, they survive by eating other fish. In other lakes where Sovkafish exist, their populations are limited by Dragonfish, which like to feed on Sovkafish. Which of the following, if true, most effectively challenges the conclusion that releasing 100 Dragonfish into Chilliwack Lake will allow the Gigafish in Chilliwack Lake to survive? (watch the entire video here)
  • 32. Dissecting an Argument Until recently, the only fish species living in Chilliwack Lake was the Gigafish. Last month, however, several Sovkafish were spotted in the lake. Unlike Gigafish, Sovkafish do not eat insects; instead, they survive by eating other fish. In other lakes where Sovkafish exist, their populations are limited by Dragonfish, which like to feed on Sovkafish. Which of the following, if true, most effectively challenges the conclusion that releasing 100 Dragonfish into Chilliwack Lake will allow the Gigafish in Chilliwack Lake to survive? Conclusion: Releasing Dragons will let Gigas live + Premise: Giga were only fish in lake Premise: Sovkas now in lake Premise: Sovkas eat other fish Premise: Dragons eat Sovkas Assumption: Dragons won't eat the Gigas Assumption: Dragons won't somehow jeopardize Gigas Assumption: 100 Dragons is sufficient (watch the entire video here)
  • 33. Dissecting an Argument Premise-Therefore-Conclusion test Tips for Identifying Conclusions and Premises • Watch for trigger words that indicate a conclusion • Watch for trigger words that indicate a premise therefore, thus, hence, so, implies, indicates, consequently, as a result, clearly, accordingly, infer, conclude since, because, for, due to, evidence, on the basis of, given that • Beware of common argument structures 1. Premise, premise, . . . , conclusion 2. Conclusion, premise, premise . . . 3. Conclusion in the question stem (watch the entire video here)
  • 34. Common Argument Types (watch the entire video here)
  • 35. Common Argument Types 3 Most Common Argument Types • Cause and Effect • Statistical • Analogy (watch the entire video here)
  • 36. Common Argument Types Cause and Effect Conclusion: X causes Y + Premise: Event X occurs Premise: Event Y occurs Assumption: X is the only possible cause of Y A recent study reveals that the rate of obesity is higher among senior citizens who watch more than 8 hours of television per day than among senior citizens who watch fewer than 8 hours of television per day. Therefore, obesity among senior citizens is caused by watching more than 8 hours of television per day. correlation ≠ causation Weaken • Something else causes Y • Y causes X • X and Y are coincidental Strengthen • More information • Eliminate other causes of Y (watch the entire video here)
  • 37. Common Argument Types Statistical Conclusion: Something about entire population + Premise: Information from sample Assumption: Sample represents entire population In a recent survey, participants at a Republicans- only dance competition were given a questionnaire. Most of the respondents indicated that they enjoyed singing. Therefore, it can be concluded that most Republicans are outgoing people. Weaken • Sample not representative • Conclusion doesn’t match stats • Flaw in calculations Strengthen • Sample is representative (watch the entire video here)
  • 38. Common Argument Types Analogy Conclusion: Some other similarity exists + Premise: Similarity between X and Y Premise: Similarity between X and Y Premise: Similarity between X and Y Assumption: Sharing some means sharing all Country X is a democratic, tropical country with a population of 5 million, and Country Y is a democratic, tropical country with a population of 5 million. Since Country X is experiencing widespread crop failures, Country Y must be experiencing widespread crop failures as well. Weaken • Entities less similar Strengthen • Entities even more similar (watch the entire video here)
  • 39. Common Argument Types Cause and Effect Conclusion: X causes Y + Premise: Event X occurs Premise: Event Y occurs Assumption: X is the only possible cause of Y Weaken • Something else causes Y • Y causes X • X and Y are coincidental Strengthen • More information • Eliminate other causes of Y Statistical Conclusion: Something about entire population + Premise: Information from sample Assumption: Sample represents entire population Weaken • Sample not representative • Conclusion doesn’t match stats • Flaw in calculations Strengthen • Sample is representative Analogy Conclusion: Some other similarity exists + Premise: Similarity between X and Y Premise: Similarity between X and Y Premise: Similarity between X and Y Assumption: Sharing some means sharing all Weaken • Entities less similar Strengthen • Entities even more similar (watch the entire video here)
  • 40. General Critical Reasoning Strategy (watch the entire video here)
  • 41. Question Types General Critical Reasoning Strategy 1. Weaken the Argument 2. Strengthen the Argument 3. Assumption 4. Conclusion/Inference 5. Method of Reasoning 6. Flawed Argument 7. Paradox 8. Evaluation (watch the entire video here)
  • 42. General Strategy General Critical Reasoning Strategy 1. Read the question stem to determine the question type 2. Read the passage (argument) and focus on the required information for that question type 3. Check all answer choices (watch the entire video here)
  • 43. Weaken the Argument Questions (watch the entire video here)
  • 44. Weaken the Argument Questions Conclusion Premise Premise Premise Assumption Assumption+ (A) New Premise (B) New Premise (C) New Premise (D) New Premise (E) New Premise Goal: Find the answer choice that, when added to the argument, undermines the conclusion the most. (watch the entire video here)
  • 45. Weaken the Argument Questions General Strategy 1. Read the question stem to determine the question type 2. Read the passage (argument) and focus on the required information for that question type 3. Check all answer choices (watch the entire video here)
  • 46. Weaken the Argument Questions Question stem examples Which of the new premises hurts the argument the most? • Which of the following, if true, most seriously weakens the argument above? • Which of the following, if true, would cast the most serious doubt on the researcher’s conclusion? • Which of the following, if true, most effectively challenges the conclusion that releasing 100 Dragonfish into Chilliwack Lake will allow the Gigafish in Chilliwack Lake to survive? • Which of the following, if established, most seriously calls into question the argument above? • Which of the following, if true, most seriously undermines the spokesperson’s position? (watch the entire video here)
  • 47. Weaken the Argument Questions General Strategy 1. Read the question stem to determine the question type 2. Read the passage (argument) and focus on the required information for that question type 3. Check all answer choices (watch the entire video here)
  • 48. Weaken the Argument Questions Weaken the Argument Strategy 1. Identify and summarize the conclusion 2. Identify and summarize the premises 3. Identify any assumptions 4. Check each answer choice while repeating conclusion 5. Check all answer choices Does this weaken the conclusion that...? (watch the entire video here)
  • 49. Weaken the Argument Questions Tips - Undermining an unstated assumption - Adding a new premise that hurts the conclusion 1. Look for common argument types (cause and effect, statistical, analogy) 2. Common ways to weaken an argument 3. “Weaken” does not necessarily mean “destroy” 4. Beware of answer choices that strengthen the argument 5. Do not try to disprove a premise 6. Goal: Weaken the extent to which the conclusion follows from the premises (watch the entire video here)
  • 50. Weaken the Argument Questions Weaken the Argument Strategy 1. Identify and summarize the conclusion 2. Identify and summarize the premises 3. Identify any assumptions 4. Check each answer choice while repeating conclusion 5. Check all answer choices Example conclusion: Eating carrots improves one’s eyesight Does this weaken the conclusion that...? (watch the entire video here)
  • 51. Practice Question A program instituted in a particular state allows parents to prepay their children's future college tuition at current rates. The program then pays the tuition annually for the child at any of the state's public colleges in which the child enrolls. Parents should participate in the program as a means of decreasing the cost of their children's college education. Which of the following, if true, is the most appropriate reason for parents not to participate in the program? A. The parents are unsure about which public college in the state the child will attend. B. The amount of money accumulated by putting the prepayment funds in an interest-bearing account today will be greater than the total cost of tuition for any of the public colleges when the child enrolls. C. The annual cost of tuition at the state's public colleges is expected to increase at a faster rate than the annual increase in the cost of living. D. Some of the state's public colleges are contemplating large increases in tuition next year. E. The prepayment plan would not cover the cost of room and board at any of the state's public colleges. This is a question from the GMAT® mini-test. GMAT® questions are the property of the Graduate Management Admission Council® and are used here with their permission.
  • 52. Practice Question A program instituted in a particular state allows parents to prepay their children's future college tuition at current rates. The program then pays the tuition annually for the child at any of the state's public colleges in which the child enrolls. Parents should participate in the program as a means of decreasing the cost of their children's college education. Which of the following, if true, is the most appropriate reason for parents not to participate in the program? P: Program: prepay tuition @ current rates P: Program pays tuition later C: Program is cheaper A: Today’s cost less than future cost A. The parents are unsure about which public college in the state the child will attend. B. The amount of money accumulated by putting the prepayment funds in an interest-bearing account today will be greater than the total cost of tuition for any of the public colleges when the child enrolls. C. The annual cost of tuition at the state's public colleges is expected to increase at a faster rate than the annual increase in the cost of living. D. Some of the state's public colleges are contemplating large increases in tuition next year. E. The prepayment plan would not cover the cost of room and board at any of the state's public colleges.     (watch the entire video here)
  • 53. Practice Question To attract members away from other fitness clubs in the city, FitnessTown recently launched a new marketing campaign in which each new member receives a free locker rental, free fitness consultations, and unlimited use of the tanning beds. FitnessTown’s marketing team claims that the campaign has been a huge success, since its membership has already increased 20 percent in the first two weeks of the campaign. Which of the following, if true, would best help to refute the claim of FitnessTown’s marketing team? A. Almost all of FitnessTown’s new members had never visited a fitness club before they were enticed by FitnessTown’s new campaign. B. Most of FitnessTown’s new members do not intend to use the free services offered in the campaign. C. FitnessTown’s investors hope to increase membership by 50 percent within the first month of the campaign. D. FitnessTown’s new marketing campaign is identical to its previous marketing campaigns. E. Studies show that people typically cancel their fitness club memberships within the first year.
  • 54. Practice Question To attract members away from other fitness clubs in the city, FitnessTown recently launched a new marketing campaign in which each new member receives a free locker rental, free fitness consultations, and unlimited use of the tanning beds. FitnessTown’s marketing team claims that the campaign has been a huge success, since its membership has already increased 20 percent in the first two weeks of the campaign. Which of the following, if true, would best help to refute the claim of FitnessTown’s marketing team? A. Almost all of FitnessTown’s new members had never visited a fitness club before they were enticed by FitnessTown’s new campaign. B. Most of FitnessTown’s new members do not intend to use the free services offered in the campaign. C. FitnessTown’s investors hope to increase membership by 50 percent within the first month of the campaign. D. FitnessTown’s new marketing campaign is identical to its previous marketing campaigns. E. Studies show that people typically cancel their fitness club memberships within the first year. P: Campaign = locker, consult. & tanning P: Membership up 20% in 2 wks C: FitnessTown stole others’ members A: membership increase means stolen customers.     (watch the entire video here)
  • 55. Strengthen the Argument Questions (watch the entire video here)
  • 56. Conclusion Premise Premise Premise Assumption Assumption+ (A) New Premise (B) New Premise (C) New Premise (D) New Premise (E) New Premise Goal: Find the answer choice that, when added to the argument, strengthens the conclusion the most. Strengthen the Argument Questions (watch the entire video here)
  • 57. Question stem examples Which of the new premises improves the argument the most? Strengthen the Argument Questions • Which of the following, if true, would provide the most support for the conclusion of the argument above? • Which of the following statements, if true, most strengthens the author’s argument? • Which of the following, if true, provides the best indication that Suki’s decision was logically sound? • Which of the following, if true, most strongly supports the recommendation made by the argument? (watch the entire video here)
  • 58. Does this strengthen the conclusion that...? Example conclusion: Antonio makes the world’s best spaghetti Strengthen the Argument Strategy Strengthen the Argument Questions 1. Identify and summarize the conclusion 2. Identify and summarize the premises 3. Identify any assumptions 4. Check each answer choice while repeating conclusion 5. Check all answer choices (watch the entire video here)
  • 59. Tips • Look for common argument types (cause and effect, statistical, analogy) • Common ways to strengthen an argument • The goal is not to create a perfect argument • Beware of answer choices that weaken the argument • Watch out for answer choices that support a premise but not the conclusion Strengthen the Argument Questions - Stating a previously-unstated assumption - Supporting or elaborating on an existing premise - Adding a new supporting premise Bonnie must be great chess player Avram is world-class sprinter Avram is great chess player Bonnie is world-class sprinter+ (A) Bonnie holds 100m world record (B) . (C) . (D) . (E) .  (watch the entire video here)
  • 60. Strengthen the Argument Questions Does this strengthen the conclusion that...? Strengthen the Argument Strategy 1. Identify and summarize the conclusion 2. Identify and summarize the premises 3. Identify any assumptions 4. Check each answer choice while repeating conclusion 5. Check all answer choices (watch the entire video here)
  • 61. Practice Question In the 12 years since the pretzel desalinization plant was built on the Polahoochi River, the salinity of the river has increased from 180 µS/cm to 911 µS/cm in the 10-mile stretch of river downstream from the plant. During the same 12-year period, researchers have observed a sharp decline in the average number of eggs laid by individual polafish, a species native to the Polahoochi River. Given this information, Dr. Kim hypothesizes that high salinity levels adversely affect the reproductive organs of the polafish. Which of the following, if true, would most strengthen Dr. Kim’s hypothesis? A. In the same state, many populations of polafish that are not downstream of pretzel desalinization plants have experienced a reduction in egg production. B. Prior to the construction of the pretzel desalinization plant, the salinity of the 10-mile stretch of river downstream never exceeded 180 µS/cm. C. Other species of fish in the same 10-mile stretch have experienced a sharp reduction in egg production. D. In the past 12 years, the salinity of the tributaries flowing into the 10-mile stretch of river downstream from the plant has remained below 180 µS/cm, and the polafish living in these tributaries have not experienced any decline in egg production. E. In other states, fish downstream from pretzel desalinization plants have experienced declines in egg production.
  • 62. Practice Question In the 12 years since the pretzel desalinization plant was built on the Polahoochi River, the salinity of the river has increased from 180 µS/cm to 911 µS/cm in the 10-mile stretch of river downstream from the plant. During the same 12-year period, researchers have observed a sharp decline in the average number of eggs laid by individual polafish, a species native to the Polahoochi River. Given this information, Dr. Kim hypothesizes that high salinity levels adversely affect the reproductive organs of the polafish. Which of the following, if true, would most strengthen Dr. Kim’s hypothesis? P: plant blt 12 yrs ago P: downstream: salt  P: downstream: polafish eggs  C:  salinity lowers polafish egg #s A:  salinity is only possible cause of egg  A. In the same state, many populations of polafish that are not downstream of pretzel desalinization plants have experienced a reduction in egg production. B. Prior to the construction of the pretzel desalinization plant, the salinity of the 10-mile stretch of river downstream never exceeded 180 µS/cm. C. Other species of fish in the same 10-mile stretch have experienced a sharp reduction in egg production. D. In the past 12 years, the salinity of the tributaries flowing into the 10-mile stretch of river downstream from the plant has remained below 180 µS/cm, and the polafish living in these tributaries have not experienced any decline in egg production. E. In other states, fish downstream from pretzel desalinization plants have experienced declines in egg production.     (watch the entire video here)
  • 63. Assumption Questions (watch the entire video here)
  • 64. Conclusion Premise Premise Premise Assumption Assumption+ (A) Assumption (B) Assumption (C) Assumption (D) Assumption (E) Assumption Goal: Find a necessary assumption Assumption Questions (watch the entire video here)
  • 65. Question stem examples Assumption Questions • Which of the following is an assumption on which the argument depends? • The scientist’s argument depends on the assumption that • The conclusion above follows logically if which of the following is assumed? (watch the entire video here)
  • 66. Is this assumption necessary to draw the conclusion that...? Assumption Question Strategy 1. Identify and summarize the conclusion 2. Identify and summarize the premises 3. Identify any assumptions 4. Look for one of your assumptions among the answer choices 5. Check each answer choice against the conclusion Assumption Questions Example conclusion: Hotdogs are bad for one’s health (watch the entire video here)
  • 67. Strengthen the Argument Questions Assumption Questions Assumption Questions Want information that helps the conclusion Want information that is necessary for the conclusion (watch the entire video here)
  • 68. Negation Technique Assumption Questions Basis: 1) An assumption is absolutely necessary for a conclusion to follow from the premises 2) Negating a necessary assumption will destroy the argument Juan has been practicing tennis 3 hours each day for the past 2 years. Therefore, Juan will win the city championship next month. C: J will win championship P: J practicing 3hr/day for 2 yrs + A: Nothing stops championship A: J is eligible to play A: It is not the case that J lives until championship A: J lives until championship P: J dies before championship P: J dies before championship   (watch the entire video here)
  • 69. Is this assumption necessary to draw the conclusion that...? Assumption Question Strategy 1. Identify and summarize the conclusion 2. Identify and summarize the premises 3. Identify any assumptions 4. Look for one of your assumptions among the answer choices 5. Check each answer choice against the conclusion 6. Apply Negation Technique: 7. Check all answer choices Assumption Questions Negate each answer choice and insert it into the argument. The negated answer choice that destroys the argument is the correct answer (watch the entire video here)
  • 70. Tips • Look for common argument types (cause and effect, statistical, analogy) • Look for shifts in language between premises and conclusion • Remember that arguments can have any number of assumptions Assumption Questions Juan has been practicing tennis 3 hours each day for the past 2 years. Therefore, Juan will win the city championship next month. C: J will win championship P: J practicing 3hr/day for 2 mths + A: J lives until championship A: Nothing stops championship A: J is eligible to play (watch the entire video here)
  • 71. Practice Question A. Investment firms should not restrict the ways in which portfolio managers manage their clients’ funds. B. Portfolio managers can provide above- average returns for their corporate clients only by short selling stocks. C. Short selling is a technique used primarily for corporate clients. D. Portfolio managers often used short selling techniques to provide above- average returns for corporate clients. E. Before the investment firm issued the new rules, portfolio managers were not permitted to short sell stocks. A leading investment firm has issued new rules that prevent its portfolio managers from short selling stocks for their corporate clients. One portfolio manager has concluded that this restriction prevents portfolio managers from providing above-average investment returns for their corporate clients. Which of the following is an assumption that would allow the portfolio manager’s conclusion to be properly drawn?
  • 72. Practice Question A. Investment firms should not restrict the ways in which portfolio managers manage their clients’ funds. B. Portfolio managers can provide above- average returns for their corporate clients only by short selling stocks. C. Short selling is a technique used primarily for corporate clients. D. Portfolio managers often used short selling techniques to provide above- average returns for corporate clients. E. Before the investment firm issued the new rules, portfolio managers were not permitted to short sell stocks. A leading investment firm has issued new rules that prevent its portfolio managers from short selling stocks for their corporate clients. One portfolio manager has concluded that this restriction prevents portfolio managers from providing above-average investment returns for their corporate clients. Which of the following is an assumption that would allow the portfolio manager’s conclusion to be properly drawn? A: SS necessary to make big $     P: New rules prevent SS C: Rules stop big $ for CC (watch the entire video here)
  • 73. Conclusion/Inference Questions (watch the entire video here)
  • 74. ? Premise Premise Premise+ (A) Conclusion (B) Conclusion (C) Conclusion (D) Conclusion (E) Conclusion Goal: Find conclusion that logically follows Conclusion/Inference Questions (watch the entire video here)
  • 75. Question stem examples Conclusion/Inference Questions • The statements above, if true, most strongly support which of the following conclusions? • If the statements above are true, which of the following must also be true on the basis of them? • Which of the following hypotheses receives the strongest support from the given information? • Which of the following can be logically inferred based on the statements above? Identify something that must follow from the premises Inference question = Conclusion question (watch the entire video here)
  • 76. Conclusion/Inference Questions For the past 3 days, all of Florida’s orange farms have experienced freezing temperatures. Therefore, the number of oranges harvested this year will be less than expected. Typical Conclusion (in most GMAT questions) Conclusion in a Conclusion question • Conclusion is partially supported For the past 3 days, the temperature at every Florida orange farm has not exceeded -5 degrees Celsius. The statement above, if true, most strongly supports which of the following conclusions? • Conclusion is guaranteed (A) The number of oranges harvested this year will be less than expected.  (watch the entire video here)
  • 77. Conclusion/Inference Questions For the past 3 days, all of Florida’s orange farms have experienced freezing temperatures. Therefore, the number of oranges harvested this year will be less than expected. Typical Conclusion (in most GMAT questions) Conclusion in a Conclusion question • Conclusion is partially supported For the past 3 days, the temperature at every Florida orange farm has not exceeded -5 degrees Celsius. The statement above, if true, most strongly supports which of the following conclusions? • Conclusion is guaranteed (A) The orange harvest at some Florida farms will be less than expected.  (watch the entire video here)
  • 78. Conclusion/Inference Questions For the past 3 days, all of Florida’s orange farms have experienced freezing temperatures. Therefore, the number of oranges harvested this year will be less than expected. Typical Conclusion (in most GMAT questions) Conclusion in a Conclusion question • Conclusion is partially supported For the past 3 days, the temperature at every Florida orange farm has not exceeded -5 degrees Celsius. The statement above, if true, most strongly supports which of the following conclusions? • Conclusion is guaranteed (A) At least one orange will be damaged from the freezing temperatures.  (watch the entire video here)
  • 79. Conclusion/Inference Questions For the past 3 days, all of Florida’s orange farms have experienced freezing temperatures. Therefore, the number of oranges harvested this year will be less than expected. Typical Conclusion (in most GMAT questions) Conclusion in a Conclusion question • Conclusion is partially supported For the past 3 days, the temperature at every Florida orange farm has not exceeded -5 degrees Celsius. The statement above, if true, most strongly supports which of the following conclusions? • Conclusion is guaranteed (A) For the past 3 days, not one Florida orange farm has experienced temperatures above -5 degrees Celsius.  (watch the entire video here)
  • 80. Conclusion/Inference Questions For the past 3 days, all of Florida’s orange farms have experienced freezing temperatures. Therefore, the number of oranges harvested this year will be less than expected. Typical Conclusion (in most GMAT questions) Conclusion in a Conclusion question • Conclusion is partially supported For the past 3 days, the temperature at every Florida orange farm has not exceeded -5 degrees Celsius. The statement above, if true, most strongly supports which of the following conclusions? • Conclusion is guaranteed (A) During the past 3 days, at least one Florida orange farm has experienced temperatures lower than 40 degrees Celsius.  (watch the entire video here)
  • 81. Must it be true that...? Conclusion Question Strategy 1. Identify and summarize the premises 2. Draw a conclusion that must follow 3. Look for your conclusion among the answer choices 4. Aggressively eliminate incorrect answers 5. Apply a version of the Negation Technique: 6. Check all answer choices Conclusion/Inference Questions The negated conclusion that contradicts the premises the most is probably the correct answer. (watch the entire video here)
  • 82. Tips 1. Do not stray too far from the premises 2. Look for a rewording of a premise P: Kyle enjoys chocolate P: Rome is the capital of Italy Conclusion/Inference Questions Example C: ? What conclusion can be drawn using every premise?  (watch the entire video here)
  • 83. Tips 1. Do not stray too far from the premises 2. Look for a rewording of a premise P: Kyle enjoys chocolate P: Rome is the capital of Italy Conclusion/Inference Questions Example C: The capital of Italy is Rome  (watch the entire video here)
  • 84. Tips 1. Do not stray too far from the premises 2. Look for a rewording of a premise 3. Conclusions need not involve every premise 4. Do not inject assumptions into the argument 5. Beware of answer choices that introduce new ideas/words 6. Beware of answer choices where the strength of the language does not match the strength of the language in the premises Conclusion/Inference Questions (watch the entire video here)
  • 85. Practice Question While studying Emperor Penguins in Antarctica during the coldest 3 months of the year, researchers observed that several of the penguins died. The researchers also discovered that the death rate among the larger penguins was greater than the death rate among the smaller penguins. However, the researchers’ conclusion that size was a determinant in these deaths is probably mistaken, since smaller penguins are typically younger than larger ones. If the above statements are true, which of the following can be inferred? A. Among Emperor Penguins of the same age, the smaller penguins have a greater survival rate over a 3-month period than the larger penguins do. B. Among the Emperor Penguins that survived the 3-month period, there is probably no relationship between age and size. C. Among Emperor Penguins of the same age, a difference in size may not indicate a difference in chances of survival over a 3-month period. D. Temperature does not play a role in the death rate among Emperor Penguins over a 3-month period. E. Among Emperor Penguins of the same size, age is a determinant in the survival rate over a 3-month period.
  • 86. Practice Question While studying Emperor Penguins in Antarctica during the coldest 3 months of the year, researchers observed that several of the penguins died. The researchers also discovered that the death rate among the larger penguins was greater than the death rate among the smaller penguins. However, the researchers’ conclusion that size was a determinant in these deaths is probably mistaken, since smaller penguins are typically younger than larger ones. If the above statements are true, which of the following can be inferred? P: P’s died during 3 coldest mnths P: Larger P’s more likely to die P: Resrchrs conclude size a factor P: Size probably not a factor P: Smaller P’s typically younger A. Among Emperor Penguins of the same age, the smaller penguins have a greater survival rate over a 3-month period than the larger penguins do. B. Among the Emperor Penguins that survived the 3-month period, there is probably no relationship between age and size. C. Among Emperor Penguins of the same age, a difference in size may not indicate a difference in chances of survival over a 3-month period. D. Temperature does not play a role in the death rate among Emperor Penguins over a 3-month period. E. Among Emperor Penguins of the same size, age is a determinant in the survival rate over a 3-month period.     (watch the entire video here)
  • 87. Structure Questions (watch the entire video here)
  • 88. Structure Questions • Test your understanding of the argumentative strategies employed in an argument • 3 types of Structure Questions: - Method of Reasoning - Boldface - Parallel Argument (watch the entire video here)
  • 89. (A) Description of argument (B) Description of argument (C) Description of argument (D) Description of argument (E) Description of argument Goal: Find the best description of the author’s argumentative strategy. Method of Reasoning questions Conclusion Premise Premise Premise Assumption Assumption+ Structure Questions (watch the entire video here)
  • 90. Question stem examples for Method of Reasoning questions Explain how the author presents his/her argument Structure Questions • The author’s point is made by which method of reasoning? • Which of the following strategies does Dr. Kwan use to defend his position? • In the passage, the author develops the argument by ___ • The reporter challenges the spokesperson’s position by doing which of the following? (watch the entire video here)
  • 91. Examples of answer choices for Method of Reasoning questions Structure Questions • The argument arrives at its conclusion by demonstrating the inherent problems with alternative conclusions. • The author offers a new definition of a term that is central to an opposing argument. • The argument employs circular reasoning by assuming that which it is trying to prove. The answer choices are typically generic (watch the entire video here)
  • 92. Strategy for tackling Method of Reasoning questions 1. Read the passage 2. For each sentence, ask, “What role does this play in the argument?” 3. Identify and summarize the conclusion and premises 4. Use generic language to describe the method of reasoning to yourself 5. Look for your description among the answer choices 6. Check all answer choices Structure Questions (watch the entire video here)
  • 93. (A) Role played by boldfaced portion(s) (B) Role played by boldfaced portion(s) (C) Role played by boldfaced portion(s) (D) Role played by boldfaced portion(s) (E) Role played by boldfaced portion(s) Goal: Find the best description of the role(s) played Boldface questions Conclusion Premise Premise Premise Assumption Assumption+ Structure Questions (watch the entire video here)
  • 94. Question stem examples for Boldface questions The passage contains bolded text Structure Questions • In the above argument, the portion in boldface plays which of the following roles? • In the researcher’s argument, the two portions in boldface play which of the following roles? Researcher: Two years ago, a wolf pack was relocated to Bilford Island. Although the local rabbit population has decreased drastically since the relocation, the wolves are not to blame for this decrease. Our study shows that the unprecedented number of recent rabbit deaths is due to the myxoma virus. In the above argument, the portion in boldface plays which of the following roles? (watch the entire video here)
  • 95. Strategy for tackling Boldface questions 1. Read the passage 2. For each boldfaced portion, ask, “What role does this play in the argument?” 3. Identify and summarize the conclusion and premises 4. Use generic language to describe the roles played by the boldfaced portion(s) 5. Find the answer choice that most closely matches yours 6. Check all answer choices Structure Questions (watch the entire video here)
  • 96. Tips for Boldface questions 1. Look for common roles: 2. Consider how the second bolded part is related to first bolded part 3. Beware of answer choices that are half right and half wrong Structure Questions - Concluding - Summarizing - Contradicting - Providing supporting evidence - Providing an example - Providing a counterexample - Generalizing (watch the entire video here)
  • 97. Structure Questions (A) Complete argument (B) Complete argument (C) Complete argument (D) Complete argument (E) Complete argument Goal: Find the argument that employs the most similar argumentative strategy. Parallel Argument questions Conclusion Premise Premise Premise Assumption Assumption+ (watch the entire video here)
  • 98. Structure Questions Question stem examples for Parallel Argument questions Find the argument most like the original • Which of the following arguments exhibits a pattern of reasoning most similar to the pattern of reasoning exhibited in the argument above? • Which of the following is most like the argument above in its logical structure? • The pattern of reasoning displayed above is most closely paralleled in which of the following? (watch the entire video here)
  • 99. Structure Questions Strategy for tackling Parallel Argument questions 1. Identify and summarize the conclusion and premises 2. Use generic language to describe the method of reasoning to yourself before checking the answer choices 3. Look for an argument with same structure 4. Check all answer choices (watch the entire video here)
  • 100. Structure Questions Tips for Parallel Argument questions 1. Beware of answer choices with same subject matter 2. Questions are time-consuming check your time (watch the entire video here)
  • 101. Structure Questions • Test your understanding of the argumentative strategies employed in an argument • 3 types of Structure Questions: - Method of Reasoning - Boldfaced - Parallel Argument (watch the entire video here)
  • 102. Practice Question ABC Widget’s plan to decrease employee salaries by 15% makes sense. Granted, the reduced salaries will place a financial burden on many employees. But, if the company does not decrease employee salaries, it will not have enough money to stay in business, and everyone at the company will lose his or her job. The passage employs which of the following argumentative strategies? A. It explains why the alternative course of action would not be subject to the objections raised against the proposed course of action. B. It describes an ideal situation by way of a situation that is less than ideal. C. It indirectly arrives at its conclusion by providing grounds to reject an alternative approach. D. It extrapolates the perceived outcome of the proposed plan in order to criticize the alternative course of action. E. It employs circular reasoning by assuming that which it is trying to prove.
  • 103. Practice Question A. It explains why the alternative course of action would not be subject to the objections raised against the proposed course of action. B. It describes an ideal situation by way of a situation that is less than ideal. C. It indirectly arrives at its conclusion by providing grounds to reject an alternative approach. D. It extrapolates the perceived outcome of the proposed plan in order to criticize the alternative course of action. E. It employs circular reasoning by assuming that which it is trying to prove. P: 15% cut will hurt many P: No cut  no jobs C: 15% cut makes sense Not doing X is much worse than doing X. So, we should do X.     ABC Widget’s plan to decrease employee salaries by 15% makes sense. Granted, the reduced salaries will place a financial burden on many employees. But, if the company does not decrease employee salaries, it will not have enough money to stay in business, and everyone at the company will lose his or her job. The passage employs which of the following argumentative strategies? (watch the entire video here)
  • 104. Practice Question Fred: Columbus was a great explorer because he held on to his conviction that the Earth was round in the face of overwhelming opposition. Stan: The mark of a great explorer is bravery not the adherence to a conviction. Besides, the concept of a round Earth was widely accepted in the fifteenth century, when Columbus was looking for someone to fund his voyage. In Stan’s response, the two boldface portions play which of the following roles? A. The first presents an alternative conclusion; the second provides evidence in support of that conclusion. B. The first rejects the criterion on which Fred’s argument is based; the second disputes a specific claim. C. The first presents an alternative criterion; the second describes a premise on which Stan’s conclusion relies. D. The first elaborates on Fred’s criterion; the second presents a premise on which Stan’s conclusion relies. E. The first reveals a contradiction in Fred’s argument; the second resolves that contradiction.
  • 105. Practice Question Fred: Columbus was a great explorer because he held on to his conviction that the Earth was round in the face of overwhelming opposition. Stan: The mark of a great explorer is bravery not the adherence to a conviction. Besides, the concept of a round Earth was widely accepted in the fifteenth century, when Columbus was looking for someone to fund his voyage. In Stan’s response, the two boldface portions play which of the following roles? A. The first presents an alternative conclusion; the second provides evidence in support of that conclusion. B. The first rejects the criterion on which Fred’s argument is based; the second disputes a specific claim. C. The first presents an alternative criterion; the second describes a premise on which Stan’s conclusion relies. D. The first elaborates on Fred’s criterion; the second presents a premise on which Stan’s conclusion relies. E. The first reveals a contradiction in Fred’s argument; the second resolves that contradiction. Fred P: Conviction despite opposition C: Columbus great explorer Stan 1st: Great = bravery not conviction 2nd: Round earth known Stan 1st: Refutes standard 2nd: Calls statement false     (watch the entire video here)
  • 106. Practice Question In Townville, most smokers play tennis, and most nonsmokers do not play tennis. Therefore, in Townville, most tennis players smoke. Which of the following exhibits a pattern of flawed reasoning most similar to that in the argument above? A. In Townville, most Lions Club members were born in Townville, and most of the residents who are not Lions Club members were not born in Townville. Therefore, most of the residents who were born in Townville are Lions Club members. B. In Townville, most of the people who live west of Main Street own a GPS, and most of the people who own a GPS live east of Main street. Therefore, most of the people in Townville own a GPS. C. In Townville, most cat owners own exactly one dog, and most dog owners own more than one dog. Therefore, most of the people in Townville who own more than one dog do not own any cats. D. In Townville, most tennis players play golf, but not every golfer plays tennis. Therefore, in Townville, there are more tennis players than golfers. E. In Townville, most of the houses are painted red, and most of the houses have a pool. Therefore, in Townville, most of the houses are painted red and have a pool.
  • 107. Practice Question In Townville, most smokers play tennis, and most nonsmokers do not play tennis. Therefore, in Townville, most tennis players smoke. Which of the following exhibits a pattern of flawed reasoning most similar to that in the argument above? P: S T P: ~S ~T C: T S A. In Townville, most Lions Club members were born in Townville, and most of the residents who are not Lions Club members were not born in Townville. Therefore, most of the residents who were born in Townville are Lions Club members. B. In Townville, most of the people who live west of Main Street own a GPS, and most of the people who own a GPS live east of Main street. Therefore, most of the people in Townville own a GPS. C. In Townville, most cat owners own exactly one dog, and most dog owners own more than one dog. Therefore, most of the people in Townville who own more than one dog do not own any cats. D. In Townville, most tennis players play golf, but not every golfer plays tennis. Therefore, in Townville, there are more tennis players than golfers. E. In Townville, most of the houses are painted red, and most of the houses have a pool. Therefore, in Townville, most of the houses are painted red and have a pool.     (watch the entire video here)
  • 108. Flawed Argument Questions (watch the entire video here)
  • 109. (A) Main problem (B) Main problem (C) Main problem (D) Main problem (E) Main problem Goal: Find the argument’s primary flaw Flawed Conclusion Premise Premise Premise Assumption Assumption+ Flawed Argument Questions (watch the entire video here)
  • 110. Question stem examples Identify the main problem with the argument Flawed Argument Questions • Which of the following identifies the most serious logical flaw in the argument above? • Which one of the following best identifies the error in reasoning made in the passage? • The argument is vulnerable to criticism on which one of the following grounds? • The reasoning in the argument is not sound because it fails to establish that ___ (watch the entire video here)
  • 111. 1. Identify and summarize the conclusion and premises 2. Identify any unstated assumptions 3. Determine the primary flaw 4. Look for your answer among the answer choices 5. Check all answer choices Flawed Argument Questions Strategy (watch the entire video here)
  • 112. Common Flaws Flawed Argument Questions While studying the reading levels of elementary students, researchers discovered that, on average, the longer a child’s foot, the higher his/her reading level. Therefore, foot growth increases one’s reading level. • Confusing causation with correlation (watch the entire video here)
  • 113. Common Flaws • Confusing causation with correlation • Confusing numbers with rates Flawed Argument Questions Last year, 10 people were murdered in Happyton, and 100 people were murdered in Killington. Therefore, it is much safer to live in Happyton than in Killington (watch the entire video here)
  • 114. Common Flaws • Confusing causation with correlation • Confusing numbers with rates • Conclusion mismatch Flawed Argument Questions Last year, hundreds of postal workers in Maltania suffered back injuries from carrying heavy packages at the sorting stations. To help reduce these injuries, the government should introduce a law that prohibits people from mailing packages that are over 3 feet in length. - Watch out for new words in the conclusion (watch the entire video here)
  • 115. Common Flaws • Confusing causation with correlation • Confusing numbers with rates • Conclusion mismatch Flawed Argument Questions • Extreme conclusion - Watch out for new words in the conclusion In some countries, raising the minimum wage can help boost the economy. So, if Maltania raises its minimum wage, its economy will improve. (watch the entire video here)
  • 116. Common Flaws • Confusing causation with correlation • Confusing numbers with rates • Conclusion mismatch Flawed Argument Questions • Extreme conclusion • Mistaking necessary for sufficient - Watch out for new words in the conclusion In order to be a great science teacher, one must have a deep understanding of biology. Since Terrence has a deep understanding of biology, he must be a great science teacher. (watch the entire video here)
  • 117. Common Flaws • Confusing causation with correlation • Confusing numbers with rates • Conclusion mismatch Flawed Argument Questions • Extreme conclusion • Mistaking necessary for sufficient • Guilty by association - Watch out for new words in the conclusion Sharon, Margaret and Rena are tall, Hoopton High students who play basketball. Since Maureen is a tall, Hoopton High student, she must play basketball. (watch the entire video here)
  • 118. Common Flaws • Confusing causation with correlation • Confusing numbers with rates • Conclusion mismatch Flawed Argument Questions • Extreme conclusion • Mistaking necessary for sufficient • Guilty by association • Unrepresentative sample - Watch out for new words in the conclusion Recently, a questionnaire was given to the inhabitants of Capton, Maltania’s capital city. 83% of respondents said they own one or more horses. Therefore, we can conclude that most people in Maltania own one or more horses. (watch the entire video here)
  • 119. Flawed Argument Questions 1. Identify and summarize the conclusion and premises 2. Identify any unstated assumptions 3. Determine the primary flaw 4. Look for your description among the answer choices 5. Check all answer choices Strategy (watch the entire video here)
  • 120. Practice Question A. The author assumes that the only varieties of jabberwocky are longhaired and shorthaired. B. The author does not consider what proportion of all cases of jimmylegs involves longhaired jabberwockies. C. The author calls into question the authenticity of the WJS data without providing any data to the contrary. D. The author takes no account of the relative frequency of longhaired jabberwockies within the jabberwocky population. E. The author limits the argument to an unnecessarily restrictive subset of criteria before drawing a general conclusion. Jimmylegs, a disorder of the metatarsals, affects only jabberwockies. A popular belief contends that the longhaired variety of the jabberwocky is more predisposed to contract jimmylegs than are other varieties of jabberwocky. This, however, is clearly a myth, since data collected by the World Jabberwocky Society (WJS) indicate that, of all the jabberwockies that contract jimmylegs each year, the majority of the cases involve shorthaired jabberwockies. Which of the following highlights the most serious flaw in the above argument?
  • 121. Practice Question A. The author assumes that the only varieties of jabberwocky are longhaired and shorthaired. B. The author does not consider what proportion of all cases of jimmylegs involves longhaired jabberwockies. C. The author calls into question the authenticity of the WJS data without providing any data to the contrary. D. The author takes no account of the relative frequency of longhaired jabberwockies within the jabberwocky population. E. The author limits the argument to an unnecessarily restrictive subset of criteria before drawing a general conclusion. Jimmylegs, a disorder of the metatarsals, affects only jabberwockies. A popular belief contends that the longhaired variety of the jabberwocky is more predisposed to contract jimmylegs than are other varieties of jabberwocky. This, however, is clearly a myth, since data collected by the World Jabberwocky Society (WJS) indicate that, of all the jabberwockies that contract jimmylegs each year, the majority of the cases involve shorthaired jabberwockies. Which of the following highlights the most serious flaw in the above argument? A: Pop. breakdown supports conclusion Flaw: We don’t know the population breakdown     P: J-legs = jabberwocky disorder P: Belief: longhair predisposed to j-legs P: Majority of cases are shorthair C: Longhair not more predisposed (watch the entire video here)
  • 122. Paradox Questions (watch the entire video here)
  • 123. Goal: Find premise that resolves the paradox Paradox Questions Conclusion Premise Premise Premise+ (A) New Premise (B) New Premise (C) New Premise (D) New Premise (E) New Premise (watch the entire video here)
  • 124. Question stem examples Identify something that resolves the contradictory information Paradox Questions • The paradox described above is best resolved by which of the following? • Which of the following, if true, most helps to resolve the apparent discrepancy described above? • Which of the following, if true, best explains the paradoxical outcome of Dr. Doolittle's experiment? • Which one of the following most helps to explain the apparent contradiction above? • Which one of the following, if true, most helps to explain the difference in melting points? (watch the entire video here)
  • 125. Paradox Questions Statistics show that the number of smokers in Maltania has steadily decreased over the past 10 years. However, during the same 10 years, the total amount of tobacco sold by Maltanian tobacco farmers has increased. Which of the following, if true, most helps to resolve the apparent discrepancy described above? • Looking for an “aha” premise • Not testing ability to deconstruct arguments • Several explanations: - Farmers exporting to other markets - Big increase in tobacco chewers - and more . . . (watch the entire video here)
  • 126. Does this explain why...? Paradox Question strategy 1. Identify the contradictory premises 2. Explain the paradox to yourself 3. Check the answer choices while reminding yourself of the paradox 4. Check all answer choices Paradox Questions e.g., More tobacco sold despite fewer smokers (watch the entire video here)
  • 127. Tips • Keywords: yet, however, surprisingly, nonetheless, paradoxically • Unable to identify paradox Paradox Questions solution unlikely reread passage or guess and move on • Beware of answer choices that have opposite effect Statistics show that the number of smokers in Maltania has steadily decreased over the past 10 years. However, during the same 10 years, the total amount of tobacco sold by Maltanian tobacco farmers has increased. Which of the following, if true, most helps to resolve the apparent discrepancy described above? (A) The Maltanian government has introduced stop-smoking programs across the country.  (watch the entire video here)
  • 128. Practice Question Among the mechanics at Joe’s Garage, the senior mechanics are the most adept at diagnosing engine problems. However, the itemized bills for engine repairs made during the past four years suggest otherwise. From the bills, we see that, on average, the senior mechanics took 29 minutes to correctly diagnose an engine problem, while the junior mechanics took only 22 minutes. Which of the following, if true, most helps to resolve the apparent paradox? A. At Joe’s Garage, mechanics with more than three years of experience are called senior mechanics. B. Some of the junior mechanics had experience diagnosing engine problems before they began working at Joe’s Garage. C. At Joe’s Garage, the manager typically assigns to the junior mechanics engine problems that he expects will be relatively easy to diagnose. D. During the past 4 years, most of the senior mechanics left Joe’s Garage to open their own garages. E. Mechanics who are more adept at diagnosing engine problems are faster at completing the required repair.
  • 129. Practice Question Among the mechanics at Joe’s Garage, the senior mechanics are the most adept at diagnosing engine problems. However, the itemized bills for engine repairs made during the past four years suggest otherwise. From the bills, we see that, on average, the senior mechanics took 29 minutes to correctly diagnose an engine problem, while the junior mechanics took only 22 minutes. Which of the following, if true, most helps to resolve the apparent paradox? P: Sr. mechs best at diagnosing P: 29 min for sr. mechs, and .22 min for jr. mechs Paradox: Sr. mechanics take longer despite their superior diagnostic skills. A. At Joe’s Garage, mechanics with more than three years of experience are called senior mechanics. B. Some of the junior mechanics had experience diagnosing engine problems before they began working at Joe’s Garage. C. At Joe’s Garage, the manager typically assigns to the junior mechanics engine problems that he expects will be relatively easy to diagnose. D. During the past 4 years, most of the senior mechanics left Joe’s Garage to open their own garages. E. Mechanics who are more adept at diagnosing engine problems are faster at completing the required repair.     (watch the entire video here)
  • 130. Evaluate the Conclusion Questions (watch the entire video here)
  • 131. (A) Question (B) Question (C) Question (D) Question (E) Question Conclusion Premise Premise Premise Assumption Assumption+ Evaluate the Conclusion Questions Goal: Find the question that, when answered, best helps to evaluate the conclusion. (watch the entire video here)
  • 132. Researcher: Two years ago, a wolf pack was relocated to Bilford Island. Although the local rabbit population has decreased drastically since the relocation, the wolves are not to blame for this decrease. Our study shows that the unprecedented number of recent rabbit deaths is due to the myxoma virus. Which of the following would be most relevant to investigate in order to evaluate the researcher’s conclusion? Evaluate the Conclusion Questions Conclusion: Wolves not responsible for population decrease. A) In what season was the wolf pack relocated to Bilford Island? B) What proportion of rabbits have died from the myxoma virus? C) Did the wolf pack introduce the myxoma virus to Bilford Island? D) . . . Goal: Find the question that, when answered, helps evaluate the conclusion (watch the entire video here)
  • 133. Question stem examples Identify a question that would help gauge the strength of the conclusion Evaluate the Conclusion Questions • Knowing which of the following would be most useful in evaluating the argument? • Which of the following would be most relevant to investigate in order to evaluate the researcher’s conclusion? • Clarification of which of the following issues would be most important to evaluating the spokesperson’s position? (watch the entire video here)
  • 134. Strategy 1. Identify and summarize the conclusion and premises 2. Identify any assumptions 3. Check the answer choices by providing an answer to each question and relating it to the conclusion 4. Check all answer choices Evaluate the Conclusion Questions (watch the entire video here)
  • 135. Evaluate the Conclusion Questions Goal: Find the question that, when answered, helps evaluate the conclusion. 3. Check the answer choices by providing an answer to each question and relating it to the conclusion (watch the entire video here)
  • 136. Evaluate the Conclusion Questions Conclusion: Wolves not responsible for population decrease. 3. Check the answer choices by providing an answer to each question and relating it to the conclusion A) In what season was the wolf pack relocated to Bilford Island? Researcher: Two years ago, a wolf pack was relocated to Bilford Island. Although the local rabbit population has decreased drastically since the relocation, the wolves are not to blame for this decrease. Our study shows that the unprecedented number of recent rabbit deaths is due to the myxoma virus. Which of the following would be most relevant to investigate to evaluate the researcher’s conclusion? Summer (watch the entire video here)
  • 137. Evaluate the Conclusion Questions Conclusion: Wolves not responsible for population decrease. 3. Check the answer choices by providing an answer to each question and relating it to the conclusion A) In what season was the wolf pack relocated to Bilford Island? Researcher: Two years ago, a wolf pack was relocated to Bilford Island. Although the local rabbit population has decreased drastically since the relocation, the wolves are not to blame for this decrease. Our study shows that the unprecedented number of recent rabbit deaths is due to the myxoma virus. Which of the following would be most relevant to investigate to evaluate the researcher’s conclusion? Winter (watch the entire video here)
  • 138. Evaluate the Conclusion Questions Conclusion: Wolves not responsible for population decrease. 3. Check the answer choices by providing an answer to each question and relating it to the conclusion A) In what season was the wolf pack relocated to Bilford Island? B) What proportion of rabbits have died from the myxoma virus? Researcher: Two years ago, a wolf pack was relocated to Bilford Island. Although the local rabbit population has decreased drastically since the relocation, the wolves are not to blame for this decrease. Our study shows that the unprecedented number of recent rabbit deaths is due to the myxoma virus. Which of the following would be most relevant to investigate to evaluate the researcher’s conclusion? 43%   Winter (watch the entire video here)
  • 139. Evaluate the Conclusion Questions Conclusion: Wolves not responsible for population decrease. 3. Check the answer choices by providing an answer to each question and relating it to the conclusion A) In what season was the wolf pack relocated to Bilford Island? B) What proportion of rabbits have died from the myxoma virus? C) Did the wolf pack introduce the myxoma virus to Bilford Island? Researcher: Two years ago, a wolf pack was relocated to Bilford Island. Although the local rabbit population has decreased drastically since the relocation, the wolves are not to blame for this decrease. Our study shows that the unprecedented number of recent rabbit deaths is due to the myxoma virus. Which of the following would be most relevant to investigate to evaluate the researcher’s conclusion? 43%    Winter Yes (watch the entire video here)
  • 140. Evaluate the Conclusion Questions Conclusion: Wolves not responsible for population decrease. 3. Check the answer choices by providing an answer to each question and relating it to the conclusion A) In what season was the wolf pack relocated to Bilford Island? B) What proportion of rabbits have died from the myxoma virus? C) Did the wolf pack introduce the myxoma virus to Bilford Island? Researcher: Two years ago, a wolf pack was relocated to Bilford Island. Although the local rabbit population has decreased drastically since the relocation, the wolves are not to blame for this decrease. Our study shows that the unprecedented number of recent rabbit deaths is due to the myxoma virus. Which of the following would be most relevant to investigate to evaluate the researcher’s conclusion? 43%    Winter No (watch the entire video here)
  • 141. Evaluate the Conclusion Questions Strategy 1. Identify and summarize the conclusion and premises 2. Identify any assumptions 3. Check the answer choices by providing an answer to each question and relating it to the conclusion 4. Check all answer choices (watch the entire video here)
  • 142. Practice Question Professor Gelding: The belief that our planet will experience a wave of extreme weather events over the next 100 years is nonsense. There is no evidence to support that the rate of extreme weather events is increasing. Professor Brink: To see where you are mistaken, you need only examine the data concerning Category 5 hurricanes in the Gulf of Mexico that made landfall in the past 100 years. In the first 50 years, there were only 6 such hurricanes, but in the last 50 years, there were 19 such hurricanes. Clarification of which of the following issues would be most important to an evaluation of Professor Brink’s conclusion? A. Does the rate at which Category 5 hurricanes make landfall affect the rate of other extreme weather events? B. Do the statistics concerning Category 5 hurricanes that made landfall represent extreme weather events in the Gulf of Mexico? C. During the last 100 years, what percent of hurricanes made landfall? D. How many extreme weather events occurred in the past 100 years? E. Do the Category 5 hurricanes in the Gulf of Mexico that made landfall in the past 100 years represent extreme weather events in general?
  • 143. Practice Question A. Does the rate at which Category 5 hurricanes make landfall affect the rate of other extreme weather events? B. Do the statistics concerning Category 5 hurricanes that made landfall represent extreme weather events in the Gulf of Mexico? C. During the last 100 years, what percent of hurricanes made landfall? D. How many extreme weather events occurred in the past 100 years? E. Do the Category 5 hurricanes in the Gulf of Mexico that made landfall in the past 100 years represent extreme weather events in general? A: hurricane rate characterizes extreme weather rate P: 1st 50 yrs: 6 hurricanes P: 2nd 50 yrs: 19 hurricanes C: EW rate is increasing Professor Gelding: The belief that our planet will experience a wave of extreme weather events over the next 100 years is nonsense. There is no evidence to support that the rate of extreme weather events is increasing. Professor Brink: To see where you are mistaken, you need only examine the data concerning Category 5 hurricanes in the Gulf of Mexico that made landfall in the past 100 years. In the first 50 years, there were only 6 such hurricanes, but in the last 50 years, there were 19 such hurricanes. Clarification of which of the following issues would be most important to an evaluation of Professor Brink’s conclusion?     (watch the entire video here)
  • 144. Miscellaneous Tips (watch the entire video here)
  • 145. Miscellaneous Tips • Question type frequencies • GMAT words • EXCEPT questions • Being aggressive (watch the entire video here)
  • 146. Question type frequencies Miscellaneous Tips 1. Weaken the Argument 2. Strengthen the Argument 3. Assumption 4. Conclusion/Inference 5. Method of Reasoning 6. Flawed Argument 7. Evaluation 8. Paradox (watch the entire video here)
  • 147. Question type frequencies Miscellaneous Tips 1. Weaken the Argument 2. Strengthen the Argument 3. Assumption 4. Conclusion/Inference 5. Method of Reasoning 6. Flawed Argument 7. Evaluation 8. Paradox ¾+        (watch the entire video here)
  • 148. Miscellaneous Tips GMAT words Everybody likes ice cream= a lot of people like ice cream = most people like ice cream Common usage Everybody likes ice cream= every person likes ice cream GMAT usage (watch the entire video here)
  • 149. Miscellaneous Tips GMAT words • Read words in their strongest, most literal sense all, none, everyone, no one, always, never, each every, anywhere, nowhere • Some: 1 or more Some Gigacorp employees are college graduates. Some countries in Europe are named Italy. Some of Earth’s oceans contain salt water    • Most: More than 50% Most of Earth’s oceans contain salt water  (watch the entire video here)
  • 150. Miscellaneous Tips EXCEPT questions Today, Gary did not arrive at the office until 11am. Therefore Gary will be fired. Each of the following, if true, weakens the conclusion above EXCEPT • Does not mean “strengthen” the conclusion (watch the entire video here)
  • 151. Miscellaneous Tips EXCEPT questions Each of the following people is a former President of the United States EXCEPT      If the person is a former US president eliminate it. (A) Abraham Lincoln (B) George Washington (C) Bugs Bunny (D) Jimmy Carter (E) George Bush (watch the entire video here)
  • 152. Miscellaneous Tips EXCEPT questions Today, Gary did not arrive at the office until 11am. Therefore Gary will be fired. Each of the following, if true, weakens the conclusion above EXCEPT If it weakens the conclusion, eliminate it. (A) Work begins at noon (B) Gary is self employed (C) Gary likes ice cream    (watch the entire video here)
  • 153. Miscellaneous Tips Being aggressive • Critical reasoning questions can be time consuming • Students get lost in the answer choices • Be aggressive! “What if I eliminate all answer choices?” • This will not happen often • You will have earned time to recheck answer choices Look for reasons to reject answer choices (watch the entire video here)
  • 154. Miscellaneous Tips • Question type frequencies • GMAT words • EXCEPT questions • Being aggressive (watch the entire video here)
  • 155. Practice Question For decades, countries have measured wheat field productivity using the Wheat Field Productivity Index (WFPI), which is equal to the average number of pounds of grain harvested per acre of wheat. In 2005, Maltania’s WFPI was 60 pounds per acre more than Italy’s WFPI. In 2006, Maltania’s WFPI was 75 pounds per acre more than Italy’s. Therefore, Maltania’s wheat field productivity must have increased during that period. Which of the following, if true, provides the strongest support for the conclusion above? A. Between 2005 and 2006, the number of acres of wheat planted in both Maltania and Italy increased at the same rate. B. In 2006, Maltania received 20% more sunshine than usual. C. Between 2005 and 2006, Italy’s wheat field productivity decreased by 10 pounds per acre. D. In 2006, Italy experienced its worst drought in decades. E. In 2006, Maltania began subsidizing all of its wheat farmers.
  • 156. Practice Question P: WFPI = ave lbs wheat/acre P: 2005: Maltania’s WFPI 60 more than Italy’s P: 2006: Maltania’s WFPI 75 more than Italy’s C: Maltania’s WFPI  A. Between 2005 and 2006, the number of acres of wheat planted in both Maltania and Italy increased at the same rate. B. In 2006, Maltania received 20% more sunshine than usual. C. Between 2005 and 2006, Italy’s wheat field productivity decreased by 10 pounds per acre. D. In 2006, Italy experienced its worst drought in decades. E. In 2006, Maltania began subsidizing all of its wheat farmers. For decades, countries have measured wheat field productivity using the Wheat Field Productivity Index (WFPI), which is equal to the average number of pounds of grain harvested per acre of wheat. In 2005, Maltania’s WFPI was 60 pounds per acre more than Italy’s WFPI. In 2006, Maltania’s WFPI was 75 pounds per acre more than Italy’s. Therefore, Maltania’s wheat field productivity must have increased during that period. Which of the following, if true, provides the strongest support for the conclusion above?     (watch the entire video here)
  • 157. Practice Question All of Pedro’s friends say they know someone who is a licensed circus clown. Pedro does not know any licensed circus clowns, and he is not unique among his friends in this respect. If the above statements are true, which of the following must also be true? A. Some of Pedro’s friends are lying. B. Most licensed circus clowns lie about their profession. C. Some of Pedro’s friends have friends that Pedro is not friends with. D. Pedro is a licensed circus clown. E. All of Pedro’s friends know the same licensed circus clown.
  • 158. Practice Question All of Pedro’s friends say they know someone who is a licensed circus clown. Pedro does not know any licensed circus clowns, and he is not unique among his friends in this respect. If the above statements are true, which of the following must also be true? A. Some of Pedro’s friends are lying. B. Most licensed circus clowns lie about their profession. C. Some of Pedro’s friends have friends that Pedro is not friends with. D. Pedro is a licensed circus clown. E. All of Pedro’s friends know the same licensed circus clown. P: All friends SAY they know a C P: Pedro knows zero Cs P: Some friends don’t know a C    (watch the entire video here)
  • 159. Practice Question Reporter: The number of foreigners working at US ski resorts was lower last year than it had been in previous years. Last year, several ski resorts hired more local residents than they had hired in previous years, so it is likely that the reduced number of foreign workers at US ski resorts last year was due to the decreased demand for their services. Which of the following, if true, most seriously undermines the reporter’s argument? A. All foreigners who received work visas for last year’s ski season were able to find work at US ski resorts. B. The average wait time to receive a US work visa has decreased substantially over the past three years. C. The number of ski resorts that hired foreigners last year was the same as in previous years. D. Foreigners can work for more than one ski resort in any given year. E. Recent changes to US immigration laws drastically decreased the total number of work visas issued for last year.
  • 160. Practice Question Reporter: The number of foreigners working at US ski resorts was lower last year than it had been in previous years. Last year, several ski resorts hired more local residents than they had hired in previous years, so it is likely that the reduced number of foreign workers at US ski resorts last year was due to the decreased demand for their services. Which of the following, if true, most seriously undermines the reporter’s argument? A. All foreigners who received work visas for last year’s ski season were able to find work at US ski resorts. B. The average wait time to receive a US work visa has decreased substantially over the past three years. C. The number of ski resorts that hired foreigners last year was the same as in previous years. D. Foreigners can work for more than one ski resort in any given year. E. Recent changes to US immigration laws drastically decreased the total number of work visas issued for last year. A: Nothing else caused decrease A: Fewer foreign workers did not cause increase in locals hired     P: # foreign wrkrs lower last yr P: More locals hired last yr C:  local employees caused  foreign employees (watch the entire video here)
  • 161. Practice Question After a plague of locusts attacked every crop in Baker County, Kevin Kevinson claimed that the severity of damage to his crops was due to the fish-based fertilizer he applied to his fields before planting. Which of the following investigations is most likely to yield significant information that would help evaluate Kevin’s argument? A. Comparing the value of Kevin’s crop damage to the average value of the crop damage at farms where fish-based fertilizer was not applied to the fields before planting B. Determining the extent to which fish-based fertilizer increases crop yield C. Comparing the long-term effects of Kevin’s crop damage to the long-term effects of other crop damage in Baker County D. Comparing the severity of crop damage at Kevin’s farm with the crop damage at agriculturally similar Baker County farms where fish-based fertilizer was not applied to fields before planting E. Determining the percentage of Baker County farms that applied fish-based fertilizer to their fields before planting
  • 162. A. Comparing the value of Kevin’s crop damage to the average value of the crop damage at farms where fish-based fertilizer was not applied to the fields before planting B. Determining the extent to which fish-based fertilizer increases crop yield C. Comparing the long-term effects of Kevin’s crop damage to the long-term effects of other crop damage in Baker County D. Comparing the severity of crop damage at Kevin’s farm with the crop damage at agriculturally similar Baker County farms where fish-based fertilizer was not applied to fields before planting E. Determining the percentage of Baker County farms that applied fish-based fertilizer to their fields before planting Practice Question After a plague of locusts attacked every crop in Baker County, Kevin Kevinson claimed that the severity of damage to his crops was due to the fish-based fertilizer he applied to his fields before planting. Which of the following investigations is most likely to yield significant information that would help evaluate Kevin’s argument? P: Locusts hit all crops P: Kevin applied FBF to crops C: FBF caused severity A: Nothing else caused the severity     (watch the entire video here)
  • 163. Practice Question Ayla is taller than Elan, and Brek is the same height as Diego. Since Chayna is shorter than Diego, it follows that Ayla is taller than Chayna. Each of the following, when added to the argument as an additional premise, makes the argument logically correct EXCEPT: A. Diego is shorter than Elan. B. Ayla and Diego are the same height. C. Brek is taller than Elan. D. Elan and Chayna are the same height. E. Brek is shorter than Ayla.
  • 164. Practice Question A. Diego is shorter than Elan. B. Ayla and Diego are the same height. C. Brek is taller than Elan. D. Elan and Chayna are the same height. E. Brek is shorter than Ayla. If it makes the argument correct, eliminate it. A C Premises: Conclusion: + B=D C A E     Ayla is taller than Elan, and Brek is the same height as Diego. Since Chayna is shorter than Diego, it follows that Ayla is taller than Chayna. Each of the following, when added to the argument as an additional premise, makes the argument logically correct EXCEPT: (watch the entire video here)
  • 165. Practice Question A. In the past 7 years, the CPT for king crabs has decreased by 50%. B. 8 years ago, commercial crab-fishing boats began using high-tech sonar equipment that enables them to locate crabs with much greater accuracy. C. 9 years ago, a deadly species of sea lice drove many crab species to near extinction. D. In the past 10 years, the number of commercial crab traps set each year has remained relatively constant. E. In the past 10 years, the worldwide price of snow crabs has more than doubled. To estimate changes in crab populations, biologists monitor what is known as “crabs per trap” (CPT). The CPT is the average number of crabs that commercial crab fishermen catch per crab trap. Since the current CPT for snow crabs is the same as it was 10 years ago, biologists conclude that today’s snow crab population is approximately the same as it was 10 years ago. Which of the following, if true, most seriously weakens the argument?
  • 166. Practice Question A. In the past 7 years, the CPT for king crabs has decreased by 50%. B. 8 years ago, commercial crab-fishing boats began using high-tech sonar equipment that enables them to locate crabs with much greater accuracy. C. 9 years ago, a deadly species of sea lice drove many crab species to near extinction. D. In the past 10 years, the number of commercial crab traps set each year has remained relatively constant. E. In the past 10 years, the worldwide price of snow crabs has more than doubled. To estimate changes in crab populations, biologists monitor what is known as “crabs per trap” (CPT). The CPT is the average number of crabs that commercial crab fishermen catch per crab trap. Since the current CPT for snow crabs is the same as it was 10 years ago, biologists conclude that today’s snow crab population is approximately the same as it was 10 years ago. Which of the following, if true, most seriously weakens the argument? P: CPT = ave # crabs/trap P: CPT now = 10 yrs ago C: SC pop. ≈ 10 yrs ago A: CPT reflects pop.     (watch the entire video here)
  • 167. Practice Question Identical twins have livers that are genetically identical. Scientists recently discovered that when one twin has hepachrinosis, a debilitating liver disease, the caudate lobe of the affected twin’s liver is significantly smaller than the caudate lobe of the unaffected twin’s liver. No such difference is found when neither twin has hepachrinosis. From this information, it can be concluded that hepachrinosis is caused by diminished liver capacity. Which of the following is an assumption required by the argument? A. Many diseases are caused by diminished organ capacity. B. The caudate lobe of a healthy identical twin is the same size as the caudate lobe of a healthy person who is not an identical twin. C. When two identical twins both suffer from hepachrinosis, their caudate lobes are the same size. D. The relative smallness of the caudate lobe of an affected person is not the result of medications used in the treatment of hepachrinosis. E. People who have an identical twin are no more likely to contract hepachrinosis than people who do not have an identical twin.
  • 168. Practice Question Identical twins have livers that are genetically identical. Scientists recently discovered that when one twin has hepachrinosis, a debilitating liver disease, the caudate lobe of the affected twin’s liver is significantly smaller than the caudate lobe of the unaffected twin’s liver. No such difference is found when neither twin has hepachrinosis. From this information, it can be concluded that hepachrinosis is caused by diminished liver capacity. Which of the following is an assumption required by the argument? A. Many diseases are caused by diminished organ capacity. B. The caudate lobe of a healthy identical twin is the same size as the caudate lobe of a healthy person who is not an identical twin. C. When two identical twins both suffer from hepachrinosis, their caudate lobes are the same size. D. The relative smallness of the caudate lobe of an affected person is not the result of medications used in the treatment of hepachrinosis. E. People who have an identical twin are no more likely to contract hepachrinosis than people who do not have an identical twin. P: Twins have = livers P: Lobe of affected twin is smaller P: Lobes of unaffected twins are = C: Small lobe causes H A: nothing else causes H     (watch the entire video here)
  • 169. Practice Question On average, when 100 non-pregnant women take the Pregunda home pregnancy test, 10 of them will test positive. That is, the test will indicate that the women are pregnant. Conversely, when 100 pregnant women take the Pregunda home pregnancy test, 99 of them will test positive. Therefore, if a group of women take the Pregunda home pregnancy test, the vast majority of those who test positive will be pregnant. The above argument is flawed primarily because it A. does not take into consideration the test’s margin of error B. suggests that non-pregnant women are more likely to test positive than pregnant women are C. presupposes the proportion of pregnant women in the group D. assumes that the test’s accuracy is not affected by other factors E. ignores the fact that women who do not test positive may be pregnant
  • 170. Practice Question A. does not take into consideration the test’s margin of error B. suggests that non-pregnant women are more likely to test positive than pregnant women are C. presupposes the proportion of pregnant women in the group D. assumes that the test’s accuracy is not affected by other factors E. ignores the fact that women who do not test positive may be pregnant On average, when 100 non-pregnant women take the Pregunda home pregnancy test, 10 of them will test positive. That is, the test will indicate that the women are pregnant. Conversely, when 100 pregnant women take the Pregunda home pregnancy test, 99 of them will test positive. Therefore, if a group of women take the Pregunda home pregnancy test, the vast majority of those who test positive will be pregnant. The above argument is flawed primarily because it A: Test accuracy is constant     P: 10% of NP test positive P: 99% of P test positive C: Group  majority of positive are pregnant (watch the entire video here)
  • 171. Practice Question Jesse: The H1 vaccine prevents people from contracting thramboxia, a disease that can cause short-term paralysis. While thramboxia is an unpleasant disease with paralysis lasting up to 3 months, the H1 vaccine is not worth getting since hundreds of people die each year from the H1 vaccination. Harjit: But thousands of people die each year as a direct result of contracting thramboxia. So, in addition to preventing thramboxia, the H1 vaccine prevents deaths. Harjit responds to Jesse by A. questioning the truth of a statement on which Jesse’s conclusion is based B. suggesting that Jesse’s argument overlooks an important consequence C. noting that Jesse’s argument contradicts that which he is trying to prove D. reinforcing Jesse’s conclusion before suggesting a new conclusion E. citing evidence that disproves the evidence cited by Jesse in drawing his conclusion
  • 172. Practice Question Jesse: The H1 vaccine prevents people from contracting thramboxia, a disease that can cause short-term paralysis. While thramboxia is an unpleasant disease with paralysis lasting up to 3 months, the H1 vaccine is not worth getting since hundreds of people die each year from the H1 vaccination. Harjit: But thousands of people die each year as a direct result of contracting thramboxia. So, in addition to preventing thramboxia, the H1 vaccine prevents deaths. Harjit responds to Jesse by A. questioning the truth of a statement on which Jesse’s conclusion is based B. suggesting that Jesse’s argument overlooks an important consequence C. noting that Jesse’s argument contradicts that which he is trying to prove D. reinforcing Jesse’s conclusion before suggesting a new conclusion E. citing evidence that disproves the evidence cited by Jesse in drawing his conclusion P: H1 prevents T P: T can cause paralysis P: 100s die from H1 C: Don’t get H1 vaccine P: 1000’s die from T P: H1 prevents T & deaths C: Get H1 vaccine There’s also Y. My option accomplishes 2 things     (watch the entire video here)